Tag Archives: 2018

Đề thi và đáp án kì thi chọn đội tuyển thi Quốc gia trường Phổ thông Năng khiếu năm học 2018 – 2019

ĐỀ THI

Ngày thi thứ nhất

Bài 1. Cho số nguyên $a>1$. Tìm giá trị lớn nhất của số thực $d$ sao cho tồn tại một cấp số cộng có công sai $d$, số hạng đầu tiên là $a$ và có đúng hai trong các số $a^2, a^3, a^4, a^5$ là những số hạng của cấp số cộng đó.

Bài 2. Cho $n$ số thực $x_1, x_2, \ldots, x_n$. Với mỗi $i \in{1,2, \ldots, n}$, gọi $a_i$ là số các chỉ số $j$ mà $\left|x_i-x_j\right| \leq 1$ và $b_i$ là số các chỉ số $j$ mà $\left|x_i-x_j\right| \leq 2$ ( $i$ và $j$ có thể bằng).

(a) Chứng minh rằng tồn tại $i$ để $b_i \leq 3 a_i$.

(b) Gọi $A$ là số cặp $(i, j)$ có thứ tự mà $\left|x_i-x_j\right| \leq 1$ và $B$ là số cặp $(i, j)$ có thứ tự mà $\left|x_i-x_j\right| \leq 2$ ( $i$ và $j$ có thể bằng nhau). Chứng minh rằng $B \leq 3 A$.

Bài 3. Cho $p$ là số tự nhiên. Xét phương trình nghiệm nguyên $x^3+x+p=y^2$.

(a) Tìm số nguyên tố $p$ nhỏ nhất dạng $4 k+1$ sao cho phương trình có nghiệm.

(b) Chứng minh rằng nếu $p$ là số chính phương thì phương trình trên có nghiệm nguyên dương.

Bài 4. Cho tam giác $A B C$ nhọn nội tiếp đường tròn $(O)$ với $B, C$ cố định và $A$ di động trên $(O)$. $D$ là trung điểm $B C$. Trên $A B$ lấy các điểm $M, P$ và trên $A C$ lấy các điểm $N, Q$ sao cho $D A=D P=D Q$, dồng thời $D M \perp A C, D N \perp A B$.

(a) Chứng minh rằng các điểm $M, N, P, Q$ cùng thuộc một đường tròn $(\mathcal{C})$ và $(\mathcal{C})$ luôn đi qua một điểm cố định.

(b) Chứng minh rằng tâm của $(\mathcal{C})$ luôn thuộc một đường tròn cố định.

 

Ngày thi thứ hai

Bài 5. Cho số thực $a \neq 0$. Tìm giới hạn của dãy số $\left(u_n\right)$ thoả mãn:

$\quad\quad\quad\quad\quad\quad\quad\quad u_1=0, u_{n+1}\left(u_n+a\right)=a+1, \forall n \in \mathbb{N}^*$

Bài 6. Tìm tất cả các hàm số $f: \mathbb{R}^{+} \rightarrow \mathbb{R}^{+}$thoả mãn điều kiện:

$\quad\quad\quad\quad f\left(x f\left(y^2\right)-y f\left(x^2\right)\right)=(y-x) f(x y) \forall x, y \in \mathbb{R}^{+}, x>y$

Bài 7. Cho $n=2018.2019$. Gọi $A$ là tập hợp các bộ $\left(a_1, a_2, \ldots, a_n\right)$ có thứ tự thoả mãn điều kiện $a_i \in{0,1} \forall i \in{1,2, \ldots, n}$ và $\sum_{i=1}^n a_i=2018^2$. Có bao nhiêu bộ $\left(a_1, a_2, \ldots, a_n\right)$ từ $A$ để $\sum_{i=1}^k a_i \geq \frac{a}{2}$ và $\sum_{i=n-k+1}^n a_i \geq \frac{k}{2} \forall k \in{1,2, \ldots, n}$ ?

Bài 8. Đường tròn $(\mathcal{C})$ tâm $I$ nội tiếp tam giác $A B C$ và tiếp xúc với các cạnh $A B, A C$ tại $E, F$. $A M, A N$ là các đường phân giác trong, phân giác ngoài của góc $\angle B A C(M, N$ nằm trên $B C)$. Gọi $d_M, d_N$ lần lượt là các tiếp tuyến của $(\mathcal{C})$ qua $M, N$ và khác $B C$.

(a) Chứng minh rằng $d_M, d_N, E F$ đồng quy tại điểm $D$.

(b) Lấy trên $A B, A C$ các điểm $P, Q$ thoả mãn $D P|A C, D Q| A B$. Gọi $R, S$ là trung điểm của $D E, D F$. Chứng minh rằng $I$ thuộc đường thẳng qua các trực tâm của hai tam giác $D P S, D Q R$.

 

LỜI GIẢI

Ngày thi thứ nhất

Bài 1. Cho số nguyên $a>1$. Tìm giá trị lớn nhất của số thực $d$ sao cho tồn tại một cấp số cộng có công sai $d$, số hạng đầu tiên là $a$ và có đúng hai trong các số $a^2, a^3, a^4, a^5$ là những số hạng của cấp số cônng đó.

Lời giải: Trước hết, ta chứng minh rằng $d=a^3-a$ thoả mãn điều kiện. Thật vậy, xét cấp số cộng có số hạng đầu là $a$ và công sai là $d=a^3-a$ thì

$\quad\quad\quad\quad\quad\quad\quad\quad\quad\quad \left\{\begin{array}{l}a^3=a+\left(a^3-a\right) \\ a^5=a+\left(a^3-a\right)\left(a^2+1\right)\end{array} .\right.$

Do đó $a^3, a^5$ cùng thuộc cấp số cộng có công sai $d=a^3-a$.

Giả sử rằng tồn tại giá trị $d>a^3-a$ thoả mãn điều kiện bài toán. Khi đó:

$\quad\quad\quad\quad\quad\quad\quad\quad\quad\quad a+d>a+a^3-a=a^3$

Dẫn đến hai số hạng thuộc cấp số cộng phải là $a^4$ và $a^5$. Lại để ý rằng $a>1$ nên có $a<a^4<a^5$, kết hợp lại thì phải tồn tại hai số nguyên dương $k<l$ sao cho:

$\quad\quad\quad\quad\quad\quad\quad\quad\quad\quad \left\{\begin{array}{l}a^4=a+k d \\ a^5=a+l d\end{array}\right.$

Từ đó $a(a+k d)=a+l d$ hay $d(l-a k)=a^2-a>0$.

Chú ý rằng ta có $d>0$ nên $l-a k>0$, hơn nữa $l-a k \in \mathbb{Z}$ nên $l-a k \geq 1$. Điều này dẫn đến $a^2-a \geq d>a^3-a$, vô lý do $a>1$.

Vậy giá trị lớn nhất của $d$ là $\max d=a^3-a$.

Bài 2. Cho $n$ số thực $x_1, x_2, \ldots, x_n$. Với mỗi $i \in{1,2, \ldots, n}$, gọi $a_i$ là số các chỉ số $j$ mà $\left|x_i-x_j\right| \leq 1$ và $b_i$ là số các chỉ số $j$ mà $\left|x_i-x_j\right| \leq 2(i$ và $j$ có thể bằng nhau).

(a) Chứng minh rằng tồn tại $i$ dể $b_i \leq 3 a_i$.

(b) Gọi $A$ là số cặp $(i, j)$ có thứ tự mà $\left|x_i-x_j\right| \leq 1$ và $B$ là số cặp $(i, j)$ có thứ tự mà $\left|x_i-x_j\right| \leq 2$ ( $i$ và $j$ có thể bằng nhau). Chứng minh rằng $B \leq 3 A$.

Lời giải . (a) Không mất tính tổng quát, giả sử $x_1 \leq x_2 \leq \ldots \leq x_n$.

Xét $k=\max [a_1, a_2, \ldots, a_n]$ và $a_i=k$, khi đó tồn tại $k$ số trong dãy là:

$\quad\quad\quad\quad x_u \leq x_{u+1} \leq \ldots \leq x_i \leq \ldots \leq x_v \text { với }\left|x_u-x_i\right|,\left|x_v-x_i\right| \leq 1 .$

Ngoài ra vì tính lớn nhất của $k$ nên $\left|x_{u-1}-x_i\right|>1,\left|x_{v+1}-x_i\right|>1$.

Trong $\left[x_u, x_v\right]$, có đúng $k$ chỉ số $j$ để $\left|x_j-x_i\right| \leq 1<2$. Còn trước $x_u$, xét hai số $x_r, x_s$ sao cho $x_r \leq x_s$ và $\left|x_r-x_i\right| \leq 2,\left|x_s-x_i\right| \leq 2$ thì:

$\quad\quad\quad\quad \left|x_r-x_s\right|=x_s-x_r=\left(x_i-x_r\right)-\left(x_i-x_s\right)<2-1=1$

nên sẽ có không quá $k$ số $j$ để $\left|x_j-x_i\right| \leq 2$ vì nếu ngược lại, sẽ có nhiều hơn $k$ số liên tiếp trong dãy cách nhau không quá 1 đơn vị, mâu thuẫn với tính lớn nhất của $k$. Tương tự với các số sau $x_v$, vì thế nên $b_i \leq 3 k$ kéo theo $b_i \leq 3 a_i$.

(b) Ta sẽ chứng minh bằng quy nạp theo $n$.

Với $n=1$ rõ ràng $A=B=1$ nên khẳng định hiển nhiên đúng. Giả sử kết quả đúng với $n \geq 1$, ta sẽ chứng minh nó cũng đúng với $n+1$.

Xét dãy số thực $T=\left(x_1, x_2, \ldots, x_{n+1}\right)$ bất kỳ và giả sử $x_1 \leq x_2 \leq \ldots \leq x_{n+1}$. Ký hiệu $A_T, B_T$ là số cặp có thứ tự các chỉ số $(i, j)$ tương ứng với định nghĩa của đề bài. Giả sử $k \geq 1$ là số lượng lớn nhất các số của $T$ được chứa trong một đoạn độ dài bằng 2 nào đó.

Gọi $x_i$ là số cuối cùng của dãy mà trong đoạn $\left[x_i-1, x_i+1\right]$ có chứa đúng $k$ số (kể cả $x_i$ ). Gọi $T^{\prime}$ là dãy mới sau khi bỏ $x_i$ đi. Khi đó, số lượng các số thuộc $T^{\prime}$ có trong $\left[x_i-1, x_i+1\right]$ là $k-1$, ngoài ra $x_i$ đã bị bỏ đi thuộc về đúng $2 k-1$ cặp của $A_T$.

Do đó: $A_T=A_{T^{\prime}}+2 k-1$.

Ta viết lại như sau

$\quad\quad\quad\quad \left[x_i-2 ; x_i+2\right]=\left[x_i-2 ; x_i-1\right] \cup\left[x_i-1 ; x_i+1\right] \cup\left[x_i+1 ; x_i+2\right]$

Trừ đoạn ở giữa thì hai đoạn đầu và cuối chứa tối đa $k$ phần tử của $T$. Hơn nữa, do định nghĩa số $x_i$ nên trong đoạn $\left[x_i+1 ; x_i+2\right]$ có tối đa $k-1$ phần tử của $T$. Từ đó có tối đa:

$\quad\quad\quad\quad\quad\quad\quad\quad\quad\quad\quad\quad 2(k-1)+k=3 k-2$

phần tử của $T$ (không tính $x_i$ ) thuộc $\left[x_i-2 ; x_i+2\right]$. Dẫn đến:

$\quad\quad\quad\quad\quad\quad B_T \leq 2(3 k-2)+1+B_{T^{\prime}}=3(2 k-1)+B_{T^{\prime}}$

Áp dụng giả thiết quy nạp, ta có $B_{T^{\prime}}<3 A_{T^{\prime}}$ nên từ các điều trên thì:

$\quad\quad\quad\quad B_T \leq 3(2 k-1)+B_{T^{\prime}}<3(2 k-1)+3 A_{T^{\prime}}=3\left(A_{T^{\prime}}+2 k-1\right)=3 A_T .$

Theo nguyên lý quy nạp, bài toán cũng đúng với $n+1$.

Vậy bài toán được chứng minh hoàn toàn.

Nhận xét. Bài toán này thật ra liên quan đến phương pháp xác suất trong tổ hợp, có thể xem tại quyển “The Probabilistic Method” của GS. Noga Alon. Ta xét một lời giải khác như sau:

(a) Chọn $i$ sao cho số các chỉ số $j$ để $\left|x_i-x_j\right| \leq 1$ là lớn nhất. Khi đó, số lượng chỉ số $j$ sao cho $x_j \in\left(x_i+1, x_i+2\right]$ tối đa là $a_i$, vì nếu không thì tồn tại $j$ để $a_j>a_i$. Tương tự, số lượng chỉ số $j$ sao cho $x_j \in\left[x_i-2, x_i-1\right)$ tối đa là $a_i$.

Chú ý rằng với các chỉ số $j$ để $\left|x_i-x_j\right| \leq 2$ thì ta có điều sau:

$\quad\quad\quad\quad x_j \in\left[x_i-2, x_i-1\right) \cup\left(x_i-1, x_i+1\right) \cup\left(x_i+1, x_i+2\right]$

Số lượng các chỉ số đó chính là $b_i$, dẫn đến $b_i \leq a_i+a_i+a_i=3 a_i$. Hơn nữa, nếu đẳng thức xảy ra, ta phải có mỗi đoạn (hay nửa khoảng) ở phân hoạch trên chứa chính xác $a_i$ chỉ số $j$ của $x_j$.

(b) Bài toán hiển nhiên đúng với $n=1$. Giả sử rằng tồn tại $n>1$ để kết luận không đúng, ta chọn $n$ nhỏ nhất. Ta cũng chọn $i$ sao cho $a_i$ lớn nhất.

Gọi $A^{\prime}, B^{\prime}$ tương ứng là số cặp chỉ số $(k, l)$ mà $\left|x_k-x_l\right| \leq 1$ và $\left|x_k-x_l\right| \leq 2$, trong đó $1 \leq k, l \leq n$ và $k, l \neq i$. Vì $n$ là phản ví dụ nhỏ nhất nên $B^{\prime} \leq 3 A^{\prime}$.

Các cặp chỉ số $(k, l)$ mà $k=i$ hoặc $l=i$ và $\left|x_k-x_l\right| \leq 1$ đều phải có dạng $(k, i)$ hoặc $(i, k)$ trong đó $k \neq i$ và $(i, i)$. Có tổng cộng $2\left(a_i-1\right)+1$ cặp như thế nên $A=A^{\prime}+2\left(a_i-1\right)+1$.

Tương tự thì $B=B^{\prime}+2\left(b_i-1\right)+1$. Do đó nếu $b_i \leq 3 a_i-1$ thì:

$\quad\quad\quad\quad B=B^{\prime}+2 b_i-1 \leq 3 A^{\prime}+2\left(3 a_i-1\right)-1=3\left(A^{\prime}+2 a_i-1\right)=3 A$

Điều này trái với việc $n$ là phản ví dụ nhỏ nhất. Do đó $b_i \geq 3 a_i$. Theo ý (a) thì $b_i \leq 3 a_i$, từ đây phải có $b_i=3 a_i$. Hơn nữa, số lượng chỉ số $j$ để thỏa mãn $x_j \in\left[x_i-2, x_i-1\right)$ hoặc $x_j \in\left(x_i+1, x_i+2\right]$ dều phải bằng $a_i$.

Với mỗi $j, j^{\prime}$ sao cho $x_j, x_{j^{\prime}} \in\left[x_i-2, x_i-1\right)$, ta có $\left|x_j-x_{j^{\prime}}\right|<1$, dẫn đến $a_j \geq a_i$. Mặt khác $a_i$ là lớn nhất có thể nên $a_j=a_i$. Tương tự, với mỗi $j$ sao cho $x_j \in\left(x_i+1, x_i+2\right]$ thì $a_j=a_i$. Như vậy với mọi $j$ sao cho $1<\left|x_i-x_j\right| \leq 2$ thì $a_j=a_i$. Cũng với cách chọn chỉ số $j$ đó, lập luận tương tự như những ý trên, ta cũng phải có $b_j=3 a_j$.

Xây dựng đồ thị $\mathcal{G}$ với các đỉnh được đánh số là $1,2, \ldots, n$ sao cho cặp đỉnh $(k, l)$ kề nhau khi và chỉ khi $1<\left|x_k-x_l\right| \leq 2$. Những lập luận trên cho thấy mọi đỉnh $j$ mà tồn tại một đường đi từ $i$ đến $j$ đều phải thỏa mãn $a_j=a_i$ và $b_j=3 a_j$. Gọi $\mathcal{X}$ là tập hợp tất cả các đỉnh $j$ sao cho tồn tại một đường đi từ $i$ dến $j$ trong $\mathcal{G}$. Đặt $\mathcal{Y}={1,2, \ldots, n} \backslash \mathcal{X}(\mathcal{Y}$ có thể rỗng $)$.

Bây giờ, gọi $A_y, B_y$ tương ứng là số cặp chỉ số $(k, l)$ có tính thứ tự, có thể bằng nhau mà $\left|x_k-x_l\right| \leq 1$ và $\left|x_k-x_l\right| \leq 2$, trong đó $k, l \in \mathcal{Y}$. Chú ý rằng $A_{\mathcal{Y}}=B_{\mathcal{Y}}=0$ nếu $\mathcal{Y}=\emptyset$. Bởi $n$ là phản ví dụ nhỏ nhất, ta phải có $B_{\mathcal{Y}} \leq 3 A_{\mathcal{Y}}$. Ta gọi $a_{y, k}$ và $b_{y, k}$ tương ứng là số chỉ số $j \in \mathcal{Y}$ mà $\left|x_j-x_k\right| \leq 1$ và $\left|x_j-x_k\right| \leq 2$. Định nghĩa tương tự $a_{\mathcal{X}, k}$ và $b_{\mathcal{X}, k}$.

Với mọi $k \in \mathcal{Y}$, dễ thấy $k$ không kề bất cứ đỉnh nào trong $\mathcal{X}$, vì vậy ta có được $b_{\mathcal{X}, k}=0$ và $b_k=b_{\mathcal{Y}, k}+a_{\mathcal{X}, k}$. Từ đây dẫn đến đẳng thức sau:

$\quad\quad\quad\quad\quad\quad B=\sum_{k \in \mathcal{X}} b_k+\sum_{k \in \mathcal{Y}} b_k=3 \sum_{k \in \mathcal{X}} a_k+\sum_{k \in \mathcal{Y}}\left(b_{y, k}+a_{\mathcal{X}, k}\right)$

Ta đồng thời có $\sum_{k \in \mathcal{Y}} b_{y, k}=B_{\mathcal{Y}} \leq 3 A_{\mathcal{Y}}$. Hơn nữa, ta cũng có được:

$\quad\quad A=\sum_{k \in \mathcal{X}} a_k+\sum_{k \in \mathcal{Y}} a_k=\sum_{k \in \mathcal{X}} a_k+\sum_{k \in \mathcal{Y}}\left(a_{\mathcal{Y}, k}+a_{\mathcal{X}, k}\right)=\sum_{k \in \mathbb{X}} a_k+A_{\mathcal{Y}}+\sum_{k \in \mathcal{Y}} a_{\mathcal{X}, k}$

Do đó:

$\quad\quad\quad\quad B \leq 3 A_{\mathcal{Y}}+\sum_{k \in \mathcal{Y}} a_{\mathcal{X}, k}+3 \sum_{k \in \mathcal{X}} a_k \leq 3\left(A_{\mathcal{Y}}+\sum_{k \in \mathcal{Y}} a_{\mathcal{X}, k}+\sum_{k \in \mathcal{X}} a_k\right)=3 A$

Điều này dẫn đến giả sử phản chứng là sai.

Vì vậy, với mọi số nguyên dương $n$, ta phải có $B \leq 3 A$. Bài toán kết thúc.

Bài 3. Cho $p$ là số tự nhiên. Xét phương trình nghiệm nguyên

$\quad\quad\quad\quad\quad\quad\quad\quad\quad\quad\quad\quad x^3+x+p=y^2 .$

(a) Tìm số nguyên tố $p$ nhỏ nhất dạng $4 k+1$ sao cho phương trình có nghiệm.

(b) Chứng minh rằng nếu $p$ là số chính phương thì phương trình trên có nghiệm nguyên dương.

Lời giải. (a) Các số nguyên tố có dạng $4 k+1$ là $5,13,17, \ldots$

Trước hết, ta thấy với $p=13$ thì $x^3+x+13=y^2$ có nghiệm là $(x ; y)=(4 ; 9)$. Ta sẽ chứng minh rằng phương trình $x^3+x+5=y^2$ không có nghiệm nguyên. Xét modulo 4. Có các khả năng sau xảy ra:

  • Khi $x$ chia 4 dư $0,1,2,3$, vế trái chia 4 lần lượt dư $1,3,3,3$.
  • Khi $y$ chia 4 dư $0,1,2,3$, vế phải chia 4 lần lượt dư $0,1,0,1$.

Do đó $y$ phải lẻ và $4 \mid x$. Viết biểu thức đã cho thành:

$\quad\quad\quad\quad\quad\quad\quad\quad\quad\quad (x+3)\left(x^2-3 x+10\right)=y^2+5^2$

Do $x+3 \equiv 3(\bmod 4)$ nên $x+3$ có ước nguyên tố $q \equiv 3(\bmod 4)$. Ta biết rằng với $a, b \in \mathbb{Z}$ thì $a^2+b^2$ chia hết cho số nguyên tố $q \equiv 3(\bmod 4)$ khi và chỉ khi $q \mid a$ và $q \mid b$. Từ đó thì $q \mid 5$ hay $q=5$, mâu thuẫn.

Vậy $p=13$ là số nguyên tố nhỏ nhất cần tìm.

(b) Trước hết, ta giới thiệu kết quả sau (còn gọi là định lý 4 số):

Bổ Đề. Với các số nguyên dương $a, b, c, d$ thoả mãn $a b=c d$ thì tồn tại các số nguyên dương $x, y, z, t$ sao cho $a=x y, b=z t, c=x z, d=y t$.

Chứng minh bổ đề. Đặt $k=\operatorname{gcd}(a, c)$ và viết $a=k a_1, c=k c_1$ thì rõ ràng $\operatorname{gcd}\left(a_1, c_1\right)=1$. Thay vào đề bài, ta có

$\quad\quad\quad\quad\quad\quad\quad\quad\quad\quad k a_1 b=k c_1 d \text { hay } a_1 b=c_1 d .$

Từ đây chú ý $a_1 \mid c_1 d$, nên $a_1 \mid d$, đặt $d=a_1 \ell$. Thay vào thì có $b=\ell c_1$. Từ đó, ta chọn $x=k, y=a_1, z=c_1, t=\ell$ thì có ngay điều phải chứng minh.

Quay lại bài toán, do $p$ là số chính phương nên đặt $p=a^2, a \in \mathbb{Z}$. Ta viết lại phương trình thành dạng:

$\quad\quad\quad\quad\quad\quad\quad\quad x^3+x+a^2=y^2 \text { hay } x\left(x^2+1\right)=(y-a)(y+a) .$

Áp dụng kết quả trên vào bài toán, ta thấy tồn tại các số nguyên dương $m, n, p, q$ để $x=m n, x^2+1=p q, y+a=m p, y-a=n q$. Từ đó:

$\quad\quad\quad\quad\quad\quad\quad\quad\quad\quad (m n)^2+1=p q \text { và } m p-n q=2 a \text {. }$

Xét dãy số $\left(u_n\right)$ xác định bởi $u_0=0, u_1=1, u_{n+2}=\alpha u_{n+1}+u_n$, trong đó $\alpha$ là hằng số mà ta sẽ chọn sau. Rõ ràng với mọi $n$ thì

$\quad\quad\quad\quad\quad\quad\quad\quad u_n^2-u_{n+1} u_{n-1}=(-1)^{n-1}\left(u_1^2-u_2 u_0\right)=(-1)^{n-1} .$

Khi đó, với $n$ chẵn thì $u_n^2-u_{n+1} u_{n-1}=-1$. Chọn $m n=u_{2 k}$. Ta có:

$\quad\quad\quad\quad\quad\quad\quad u_2=\alpha, u_3=\alpha^2+1, u_4=\alpha\left(\alpha^2+2\right), u_5=\alpha^4+3 \alpha^2+1$

Chọn $p=u_3, q=u_5, m n=u_4$ thì rõ ràng $(m n)^2+1=p q$. Bây giờ ta chỉ cần có được

$\quad\quad\quad\quad\quad\quad m u_3-n u_5=2 a \text { hay } m\left(\alpha^2+1\right)-n\left(\alpha^4+3 \alpha^2+1\right)=2 a .$

Từ đây chọn $\alpha=4 a^2$ và viết $m=2 a\left(\alpha^2+2\right), n=2 a\left(\alpha^4+3 \alpha^2+1\right)$ thì đẳng thức trên sẽ thoả mãn, vì

$\quad\quad\quad\quad\quad\quad\quad\quad \left(\alpha^2+1\right)\left(\alpha^2+2\right)-\left(\alpha^4+3 \alpha^2+1\right)=1 .$

Vậy phương trình có một cặp nghiệm cụ thể là

$\quad\quad (x, y)=\left(4 a^2\left(16 a^4+2\right), 2 a\left(16 a^4+2\right)\left(16 a^4+1\right)-a\right) \text { với } a=\sqrt{p} \in \mathbb{Z}^{+} .$

Bài 4. Cho tam giác $A B C$ nhọn nội tiếp đường tròn $(O)$ với $B, C$ cố định và $A$ di động trên $(O)$. $D$ là trung điểm $B C$. Trên $A B$ lấy các điểm $M, P$ và trên $A C$ lấy các điểm $N, Q$ sao cho $D A=D P=D Q$, dồng thời $D M \perp A C, D N \perp A B$.

(a) Chứng minh rằng các điểm $M, N, P, Q$ cùng thuộc một đường tròn $(\mathcal{C})$ và (C) luôn đi qua một điểm cố định.

(b) Chứng minh rằng tâm của $(\mathcal{C})$ luôn thuộc một đường tròn cố định.

Lời giải . (a) Dễ thấy tam giác $A M Q$ cân tại $M$ nên

$\quad\quad \angle D M Q=\angle D M A=90^{\circ}-\angle A=\frac{180^{\circ}-2 \angle A}{2}=\frac{180^{\circ}-\angle P D Q}{2}=\angle D P Q$

Do đó tứ giác $M P D Q$ nội tiếp. Chứng minh tương tự, ta có tứ giác $Q N D P$ nội tiếp nên $M, N, P, Q$ cùng thuộc một đường tròn $(\mathcal{C})$, và $(\mathcal{C})$ luôn đi qua điểm $D$ cố định.

(b) Gọi $K B, K C$ là hai tiếp tuyến của $(O)$. Ta có $D, K, O$ thẳng hàng, lại có:

$\quad\quad\quad\quad\quad \angle B K O=90^{\circ}-\angle B O K=90^{\circ}-\angle B A C=\angle B M D$

Từ đó tứ giác $B D K M$ nội tiếp. Để ý rằng $K D \perp B C$ nên $K M \perp A B$, hơn nữa $D N \perp A B$ nên $K M | D N$. Tương tự thì $K N | D M$. Do đó $D M K N$ là hình bình hành hay $D K, M N$ có $J$ là trung điểm chung.

Gọi $I$ là tâm của $(\mathcal{C})$ thì $I J \perp M N$ và $J L | A D$. Chú ý rằng $D$ là tâm $(A P Q)$ và cũng là trực tâm tam giác $A M N$ nên $P Q, M N$ là hai đường đối song. Đồng thời nếu $L$ là trung điểm $A D$ thì $J L$ vuông góc với đường nối hai chân đường cao từ $M, N$ của tam giác $A M N$ nên $J L \perp P Q$. Lại có $D P=D Q$ và $I P=I Q$ nên $I D \perp P Q$, do đó $J L | D I$.

Từ đây $I D L J$ là hình bình hành và $I L, D J$ có $T$ là trung điểm chung cố định. Xét phép vị tự tâm $D$ tỉ số $\frac{1}{2}$ hợp với phép đối xứng tâm $T$ thì $A \mapsto I$. Do $A$ thuộc đường tròn $(O)$ cố định nên $I$ cũng thuộc đường tròn cố định là ảnh của $(O)$ qua hợp các phép biến hình trên. Bài toán kết thúc.

Nhận xét. Bài toán này còn một hướng tiếp cận bản chất hơn như sau. Nếu gọi $A^{\prime}$ là điểm đối xứng của $A$ qua $D$ thì $K, A^{\prime}$ là hai điểm liên hợp đẳng giác trong tam giác $A B C$, từ đó đường tròn $(\mathcal{C})$ chính là đường tròn đi qua các hình chiếu của $K, A^{\prime}$ trên các cạnh tam giác $A B C$, dồng thời $I$ là trung diểm $K A^{\prime}$.

Dưới đây là một bài toán tương tự: Cho tam giác nhọn $A B C$ nội tiếp đường tròn $(O)$ có $B C$ cố định và $A$ di dộng trên $(O)$. Gọi $H$ là trực tâm tam giác và lấy điểm $E, F$ thuộc $A B, A C$ theo thứ tự đó sao cho $H$ là trung điểm $E F$.

  1. Chứng minh rằng tâm của đường tròn $(A E F)$ luôn thuộc một đường tròn cố định. Đặt là $\omega$.
  2. Giả sử $\omega$ cắt lại $(O)$ tại các điểm $X, Y$. Chứng minh rằng $X, Y, O$ thẳng hàng.

 

Ngày thi thứ hai

Bài 5. Cho số thực $a \neq 0$. Dãy số $\left(u_n\right)$ thoả mãn:

$\quad\quad\quad\quad\quad\quad\quad\quad\quad\quad u_1=0, u_{n+1}\left(u_n+a\right)=a+1 \forall n \in \mathbb{N}^*$

Tìm giới hạn của dãy số $\left(u_n\right)$.

Lời giải: Đặt $x_{n+1}=(a+1) y_n$ và $y_{n+1}=x_n+a y_n$. Ta có:

$\quad\quad\quad\quad\quad\quad\quad y_{n+2}=x_{n+1}+a y_{n+1}=a y_{n+1}+(a+1) y_n$

Đồng thời $u_n=\frac{x_n}{y_n}$. Để ý rằng $u_1=0, u_2=\frac{a+1}{a}$. Chọn $y_1=1, y_2=a$. Từ đó:

$\quad\quad\quad\quad\quad\quad\quad\quad\quad\quad y_n=\frac{(a+1)^n-(-1)^n}{a+2} \forall n \geq 1$

Công thức trên chỉ xác định với $a \neq-2$ nên xét trường hợp $a=-2$, ta có dãy

$\quad\quad\quad\quad\quad\quad\quad\quad\quad\quad \left\{\begin{array}{l}u_1=0, \\ u_{n+1}=\frac{1}{2-u_n}, n \geq 1\end{array} .\right.$

Bằng quy nạp, ta chứng minh được $u_n \in[0 ; 1)$ nên:

$\quad\quad\quad\quad\quad\quad\quad u_{n+1}-u_n=\frac{1}{2-u_n}-u_n=\frac{\left(u_n-1\right)^2}{2-u_n}>0$

Dãy $\left(u_n\right)$ tăng và bị chặn trên bởi 1 nên có giới hạn hữu hạn là $L \in(0,1)$. Giải phương trình giới hạn, ta có được $L=\frac{1}{2-L}$. Khi đó thì $L=1$.

Tiếp theo, xét $a \neq-2$, ta có:

$\quad\quad\quad\quad u_n=\frac{x_n}{y_n}=\frac{(a+1) y_{n-1}}{y_n}=\frac{(a+1)^n+(a+1)(-1)^n}{(a+1)^n-(-1)^n} \forall n \in \mathbb{N}^*$

Đặt $-(a+1)=b \in{-1 ; 1}$, ta viết lại thành:

$\quad\quad\quad\quad\quad\quad\quad\quad\quad\quad u_n=\frac{b^n-b}{b^n-1} \forall n \geq 1$

Có các khả năng sau xảy ra:

  • Nếu $b>1$ hoặc $b<-1$, tương ứng là $a<-2$ hoặc $a>0$, thì $\lim u_n=1$.
  • Nếu $-1<b<1$, tương ứng là $-2<a<0$, thì $\lim u_n=b=-(a+1)$.

Vậy ta có kết luận sau trong các trường hợp của $a$ :

  • Nếu $a \in(-2 ; 0)$ thì $\lim u_n=-(a+1)$.
  • Nếu $a \notin(-2 ; 0)$ thì $\lim u_n=-1$.

Bài 6. Tìm tất cả các hàm số $f: \mathbb{R}^{+} \rightarrow \mathbb{R}^{+}$thoả mãn diều kiện:

$\quad\quad\quad\quad\quad\quad f\left(x f\left(y^2\right)-y f\left(x^2\right)\right)=(y-x) f(x y) \forall x, y \in \mathbb{R}^{+}, x<y .$

Lời giải . Theo giả thiết thì với mọi $y>x>0$, ta đều có

$\quad\quad\quad\quad\quad\quad\quad\quad x f\left(y^2\right)-y f\left(x^2\right)>0 \Rightarrow \frac{f\left(y^2\right)}{f\left(x^2\right)}>\frac{y}{x}>1 .$

Do đó,

$\quad\quad\quad\quad\quad\quad\quad\quad y^2>x^2 \Leftrightarrow y>x \Leftrightarrow f\left(y^2\right)>f\left(x^2\right)$

nên hàm $f$ dã cho đồng biến trên $\mathbb{R}^{+}$. Trong đề bài, thay $y=x+1$, ta có

$\quad\quad\quad\quad\quad\quad\quad f\left(x f\left((x+1)^2\right)-(x+1) f\left(x^2\right)\right)=f(x(x+1))$

hay

$\quad\quad\quad\quad\quad\quad\quad\quad x f\left((x+1)^2\right)-(x+1) f\left(x^2\right)=x(x+1) $

$\quad\quad\quad\quad\quad\quad\quad\quad \Leftrightarrow \frac{f\left((x+1)^2\right)}{x+1}=\frac{f\left(x^2\right)}{x}+1, \forall x>0$

Thực hiện thao tác này nhiều lần, ta có

$\quad\quad\quad\quad\quad\quad\quad\quad \frac{f\left((x+n)^2\right)}{x+n}=\frac{f\left(x^2\right)}{x}+n, \forall x>0, n \in \mathbb{Z}^{+}$

hay

$\quad\quad\quad\quad\quad\quad\quad\quad x f\left((x+n)^2\right)-(x+n) f\left(x^2\right)=n x(x+n) .$

Trong dề bài, thay $y=x+n$, ta có

$\quad\quad\quad\quad\quad\quad f\left(x f\left((x+n)^2\right)-(x+n) f\left(x^2\right)\right)=n f(x(x+n)) $

$\quad\quad\quad\quad\quad\quad \Leftrightarrow f(n x(x+n))=n f(x(x+n)) .$

Với mọi $n \in \mathbb{Z}^{+}, y>0$, ta luôn chọn được $x>0$ để $x(x+n)=y$ nên ta có

$\quad\quad\quad\quad\quad\quad\quad\quad\quad\quad f(n y)=n f(y), \forall n \in \mathbb{Z}^{+}, y \in \mathbb{R}^{+} .$

Đặt $f(1)=a>0$, với mọi $n \in \mathbb{Z}^{+}$, cho $y=\frac{1}{n}$, suy ra

$\quad\quad\quad\quad\quad\quad\quad\quad\quad\quad f(1)=n f\left(\frac{1}{n}\right) \Rightarrow f\left(\frac{1}{n}\right)=\frac{a}{n} .$

Do đó,

$\quad\quad\quad\quad\quad\quad\quad\quad f\left(\frac{n}{m}\right)=n f\left(\frac{1}{m}\right)=\frac{n}{m} a, \forall m, n \in \mathbb{Z}^{+}$

hay $f(x)=a x, \forall x \in \mathbb{Q}^{+}$. Với mọi số thực $x_0>0$, chọn hai dãy số hữu tỷ $\left(a_n\right),\left(b_n\right)$ sao cho $a_n<x_0<b_n$ và $\lim a_n=\lim b_n=x_0$. Rõ ràng

$\quad\quad\quad\quad\quad\quad f\left(a_n\right)<f\left(x_0\right)<f\left(b_n\right) \Rightarrow a \cdot a_n<f\left(x_0\right)<a \cdot b_n,$

nên cho $n \rightarrow+\infty$, ta có $f\left(x_0\right)=a x_0$. Do đó, với mọi số thực $x>0$ thì $f(x)=a x$. Thay vào biểu thức đã cho, ta có

$\quad\quad\quad\quad\quad \left\{\begin{array}{l}f\left(x f\left(y^2\right)-y f\left(x^2\right)\right)=a^2\left(x y^2-x^2 y\right)=a^2(y-x) x y \\ (y-x) f(x y)=a(y-x) x y\end{array}\right.$

nên $a=1$. Vậy tất cả các hàm số cần tìm là $f(x)=x, \forall x>0$.

Nhận xét. Có một điều đáng chú ý ở bài toán này là việc từ giả thiết, ta phải ngầm hiểu rằng $x f\left(y^2\right)-y f\left(x^2\right)>0$ với mọi cặp số dương $x<y$. Ta có thể thêm tường minh điều kiện đó vào đề bài cho rõ. Tuy nhiên, nếu thêm theo kiểu như sau thì sẽ có một chút vấn đề phát sinh:

Tìm tất cả các hàm số $f: \mathbb{R}^{+} \rightarrow \mathbb{R}^{+}$thoả mãn với mọi cặp số dương $x<y$, nếu $x f\left(y^2\right)-y f\left(x^2\right)>0$ thì

$\quad\quad\quad\quad\quad\quad\quad\quad\quad\quad f()=(y-x) f(x y) \forall x, y \in \mathbb{R}^{+}, x<y .$

Khi đó, ta có thể nhận thêm một hàm số thỏa mãn nữa là $f(x)=\sqrt{x}$. Lý do là vì với mọi cặp số $y>x>0$, ta đều có $x f\left(y^2\right)-y f\left(x^2\right)=0$, mà vì thế, điều kiện “nếu” ở trên là sai nên mệnh đề kéo theo là đúng.

Bài 7. Cho $n=2018.2019$. Gọi $A$ là tập hợp các bộ $\left(a_1, a_2, \ldots, a_n\right)$ có thứ tự thoả mãn điều kiện $a_i \in{0,1} \forall i \in{1,2, \ldots, n}$ và $\sum_{i=1}^n a_i=2018^2$.

Có bao nhiêu bộ $\left(a_1, a_2, \ldots, a_n\right)$ từ $A$ dể:

$\quad\quad\quad\quad\quad \sum_{i=1}^k a_i \geq \frac{a}{2} \text { và } \sum_{i=n-k+1}^n a_i \geq \frac{k}{2} \forall k \in{1,2, \ldots, n}$

Lời giải. Ta giải bài toán tổng quát khi thay 2018 bởi $m \in \mathbb{Z}^{+}$. Bài toán đã cho tương đương với bài toán sau:

Trong hệ trục tọa độ Oxy, xét lưới điểm nguyên trong hình chũ nhật có đỉnh dưới bên trái là $O(0 ; 0)$ và dỉnh trên bên phải là $A\left(m^2 ; m\right)$. Dặt $B(m ; m)$ và $C\left(m^2-m ; 0\right)$, hỏi có bao nhiêu đương đi tù̀ $O \rightarrow A$ sao cho mỗi bước, ta đi sang phải hoặc lên trên 1 đơn vị, gọi là đương đi đơn, và không vượt lên trên $O B$ cũng nhu không xuống dưới $A C$ ?

Ở đây, các số $0 ; 1$ tương ứng với các bước đi lên trên, các bước đi sang phải; còn điều kiện tổng $k$ số đầu và tổng $k$ số cuối không nhỏ hơn $\frac{k}{2}$ tương ứng với số lượng bước đi lên không vượt quá số lượng bước đi sang phải. Để thuận tiện, ta gọi đường đi cắt $d$ nếu nó có các phần nằm về cả hai phía của $d$. Trước hết, ta sẽ chứng minh bổ đề sau:

Bổ Đề. Số đường đi đơn từ $O \rightarrow A(m ; n)$, có cắt đường thẳng $y=x$, là $C_{m+n}^{m+1}$.

Thật vậy, Xét đường thẳng $(d): y=x+1$, rõ ràng các đường đi đơn cắt $y=x$ dều sẽ có điểm chung với đường thẳng $(d)$ này. Tại các điểm chung đó, ta thực hiện đối xứng trục để được một đường đi mới xuất phát từ $O \rightarrow A^{\prime}(n-1, m+1)$.

Trong hình trên, đường cũ là đứt nét, còn đường mới là liền nét. Rõ ràng phép đối xứng trục trên là song ánh, biến các đường cần tìm (cắt $y=x$ ), thành các đường từ $O \rightarrow A^{\prime}$; do đó, số lượng đường cần tìm là $C_{m+n}^{n-1}$.

Trở lại bài toán,

Số đường đi đơn từ $O \rightarrow A\left(m^2 ; m\right)$ là $C_{m^2+m}^m$ vì nó bằng số cách chọn $m$ lần đi lên trong tổng số $m^2+m$ lần di chuyển, trong đó số đường đi cắt $O B$ bằng số đường đi cắt $A C$ và bằng $C_{m^2+m}^{m-1}$ (theo bổ đề).

Do đó, ta chỉ cần tìm số đường đi cắt cả $O B, A C$ với ý tưởng đối xứng hai lần đã dùng để chứng minh bổ đề.

Đầu tiên, ta thực hiện đối xứng qua đường thẳng $y=x+1$; khi đó, các đường đi đơn sẽ xuất phát từ $O \rightarrow A^{\prime}\left(m-1 ; m^2+1\right)$. Do các đường ban đầu còn vượt qua $A C$ nên các đường mới phải cắt thêm $y=x+m^2-m+3$. Tiếp tục đối xứng qua đường thẳng này, ta đưa về đếm số đường đi đơn từ $O \rightarrow A^{\prime \prime}\left(m-2, m^2+2\right)$. Suy ra số đường đi trong trường hợp này là $C_{m^2+m}^{m-2}$. Vậy theo nguyên lý bù trừ, kết quả cần tìm sẽ là

$\quad\quad\quad\quad\quad\quad\quad\quad\quad\quad C_{m^2+m}^m-2 C_{m^2+m}^{m-1}+C_{m^2+m}^{m-2} .$

Thay $m=2018$, ta có số lượng đường đi, cũng chính là số bộ thỏa mãn đề bài.

Nhận xét. Dưới đây là một số kết quả tương tự về đường đi đơn trong đề bài

$1$. Số đường đi đơn từ $(0 ; 0) \rightarrow(m ; n)$ mà không có điểm chung với $y=x$ là

$\quad\quad\quad\quad\quad\quad\quad\quad\quad\quad\quad\quad \frac{m-n}{m+n} C_{m+n}^m .$

$2$. Số đường đi đơn từ $(0 ; 0) \rightarrow(m ; n)$ mà không vượt qua $y=x$ là

$\quad\quad\quad\quad\quad\quad\quad\quad\quad\quad\quad C_{m+n}^n-C_{m+n}^{n-1} .$

$3$. Số đường đi gồm $n$ bước mà không vượt $y=x$ là

$\quad\quad\quad\quad\quad\quad\quad\quad\quad \sum_{i=n / 2}^n \frac{n !(2 i+1-n)}{(i+1) !(n-i) !}=C_n^{[n / 2]} .$

$4$. Số đường đi đơn từ $(0 ; 0) \rightarrow(m ; n)$ mà không có điểm chung với $y=x+t$ là

$\quad\quad\quad\quad\quad\quad\quad\quad\quad\quad\quad\quad C_{m+n}^n-C_{m+n}^{m-t}$.

Bạn đọc có thể dùng phương pháp tương tự trên để giải quyết các bài toán này.

Bài 8. Đường tròn $(\mathcal{C})$ tâm $I$ nội tiếp tam giác $A B C$ và tiếp xúc với các cạnh $A B, A C$ tại $E, F$. $A M, A N$ là các đường phân giác trong, phân giác ngoài của góc $\angle B A C(M, N$ nằm trên $B C)$. Gọi $d_M, d_N$ lần lượt là các tiếp tuyến của $(\mathcal{C})$ qua $M, N$ và khác $B C$.

(a) Chứng minh rằng $d_M, d_N, E F$ dồng quy tại điểm $D$.

(b) Lấy trên $A B, A C$ các điểm $P, Q$ thoả mãn $D P|A C, D Q| A B$. Gọi $R, S$ là trung điểm của $D E, D F$. Chứng minh rằng $I$ thuộc đường thẳng qua các trực tâm của hai tam giác $D P S, D Q R$.

Lời giải. (a) Gọi $X, Y$ lần lượt là tiếp điểm của tiếp tuyến thứ hai kẻ từ $M$ dến $(I)$ và $D^{\prime}$ là tiếp điểm của $(I)$ trên $B C$. Gọi $K$ là trung điểm $E F$.

Xét trong đường tròn $(I)$ thì $E F$ là đường đối cực của $A$ và $K \in E F$ nên đối cực của $K$ sẽ đi qua $A$, mà $N A \perp I A$ nên $N A$ chính là đường đối cực của $K$.

Đường đối cực của $K$ đi qua $N$ nên đối cực của $N$, là $D^{\prime} Y$, sẽ đi qua $K$. Dễ thấy rằng $A M$ là trục đối xứng của tứ giác $D^{\prime} X E F$ nên suy ra $D^{\prime} X | E F$. Xét $D^{\prime}(E F, X Y)$, ta có có $D^{\prime} Y$ đi qua trung điểm của $E F$ và $D^{\prime} X | E F$ nên

$\quad\quad\quad\quad\quad\quad\quad\quad\quad\quad D^{\prime}(E F, X Y)=-1$

hay tứ giác $E X F Y$ điều hòa. Suy ra $M X, N Y, E F$ đồng quy. Ngoài ra ta cũng có $X, Y, A$ thẳng hàng.

(b) Dễ thấy các tam giác $P E D$ và $D Q F$ là các tam giác cân. Gọi $H_1, H_2$ lần lượt là trực tâm của tam giác $\triangle D P S, \triangle D Q R$. Ta có

$\quad\quad\quad\quad\quad\quad\quad\quad \angle P H_1 S=\angle P D F=\angle A F E=\angle P E S$

nên $E P S H_1$ là tứ giác nội tiếp. Suy ra $R H_1 \cdot R P=R S \cdot R E$. Ngoài ra,

$\quad\quad\quad\quad\quad\quad\quad\quad\quad\quad K A \cdot K I=K E \cdot K F$

nên

$\quad\quad\quad\quad\quad\quad\quad\quad\quad \frac{R P}{K A} \cdot \frac{R H_1}{K I}=\frac{R E}{K E} \cdot \frac{R S}{K F} .$

Theo định lý Thales thì $\frac{R P}{K A}=\frac{R E}{K E}$ nên $\frac{R H_1}{K I}=\frac{R S}{K F}$, mà

$\quad\quad\quad\quad\quad\quad\quad R S=R D-S D=\frac{D E-D F}{2}=\frac{E F}{2}=K F$

Suy ra $R H_1=K I$, mà $R H_1 | K I$ (do cùng vuông góc với $E F$ ) nên $I K R H_1$ là hình chữ nhật, kéo theo $I H_1 | E F$. Một cách tương tự, ta có $I H_2 | E F$ vậy nên đường thẳng $H_1 H_2$ đi qua $I$.

Nhận xét. Trong câu a, tính chất $A, X, Y$ thẳng hàng của bài toán cũng đúng khi thay $M, N$ là chân các đường phân giác bởi cặp điểm liên hợp điều hòa bất kỳ với $B, C$. Điều này có được nhờ tính chất của các đường đối cực (hoặc có thể chứng minh nhờ việc sử dụng phép chiếu trực giao các chùm điều hòa).

 

 

 

 

 

 

 

 

 

 

 

 

 

 

 

 

 

 

 

 

 

 

 

 

 

 

 

 

 

 

 

 

 

 

 

 

 

 

 

 

 

 

 

 

 

 

 

 

 

 

 

 

 

 

 

 

 

 

 

 

 

 

 

 

 

 

 

 

 

 

 

 

 

 

 

 

 

 

 

 

 

 

 

 

 

 

 

 

 

 

 

 

 

 

 

 

 

 

 

 

 

 

 

 

 

 

 

 

 

 

 

 

 

 

 

 

 

 

 

 

 

 

 

 

ĐỀ THI VÀO LỚP 10 CHUYÊN TOÁN TP.HCM NĂM 2018

Bài 1. (1 điểm )Cho $a, b, c$ là ba số thỏa điều kiện $a+b+c=0$ và $a^{2}=2(a+c+$ 1) $(a+b-1)$. Tính giá trị của biểu thức $A=a^{2}+b^{2}+c^{2}$.

Bài 2. (2 điểm)

(a) Giải phương trình: $4 \sqrt{x+3}=1+4 x+\frac{2}{x}$.

(b) Giải hệ phương trình: $\left\{\begin{array}{l}x^{2}+y^{3}=1 \\ x^{2}+y^{5}=x^{3}+y^{2}\end{array}\right.$

Bài 3. (2 điểm) Cho tam giác $A B C(A B<A C)$ vuông tại $A$ có đường cao $A H$. Gọi $E, F$ lần lượt là hình chiếu của $H$ lên $A B, A C$.

(a) Chứng minh rằng: $B E \sqrt{C H}+C F \sqrt{B H}=A H \sqrt{B C}$

(b) Gọi $D$ là điểm đối xứng của $B$ qua $H$ và gọi $O$ là trung điểm của $B C$. Đường thẳng đi qua $D$ và vuông với $B C$ cắt $A C$ tại $K$. Chứng minh rằng: $B K$ vuông góc với $A O$.

Bài 4. (1,5 điểm)

(a) Chứng minh rằng: $x^{4}-x+\frac{1}{2}>0$ với mọi số thực $x$.

(b) Cho $x, y$ là các số thực thỏa mãn điều kiện $x^{2}-x y+y^{2}=3$. Tìm giá trị lớn nhất và giá trị nhỏ nhất của biểu thức $P=x^{2}+y^{2}$.

Bài 5. (1,5 điểm) Cho tam giác $A B C$ vuông tại $A$. Gọi $M$ là trung điểm của $B C$ và $O$ là tâm đường tròn ngoại tiếp của tam giác $A M B$. Đường thẳng $A C$ cắt $(O)$ tại điểm thứ hai $K$. Đường thẳng $B K$ cắt đường tròn ngoại tam giác $A B C$ tại $L$. Các đường thẳng $C L$ và $K M$ cắt nhau tại $E$. Chứng minh rằng $E$ nằm trên đường tròn ngoại tiếp tam giác $A C M$.

Bài 6. (2 điểm) Các số nguyên dương từ 1 đến 2018 được tô màu theo nguyên tắc sau: Các số mà khi chia hết cho 24 dư 17 tô được màu xanh. Các số mà khi chia cho 40 dư 7 được tô màu đỏ. Các số còn lại được tô màu vàng.

(a) Chứng tỏ rằng không có số nào được tô cả hai màu xanh và đỏ. Hỏi có bao nhiêu số được tô màu vàng?

(b) Có bao nhiêu cặp số $(a, b)$ sao cho $a$ được tô màu xanh, $b$ được tô màu đỏ và $|a-b|$ bằng 2 ?

LỜI GIẢI

 

Bài 1. Cho $a, b, c$ là ba số thỏa điều kiện $a+b+c=0$ và $a^{2}=2(a+c+1)(a+b-1)$. Tính giá trị của biểu thức $A=a^{2}+b^{2}+c^{2}$.

Lời giải. Ta có $: a+b+c=0 \Rightarrow\left\{\begin{array}{l}a+c=-b \\ a+b=-c \\ a=-(b+c)\end{array}\right.$

Khi đó: $a^{2}=2(a+c+1)(a+b-1)=2(-b+1)(-c-1)$ $\Leftrightarrow a^{2}=2(b-1)(c+1)$

$\Leftrightarrow a^{2}=2(b c+b-c-1)$

$\Leftrightarrow(b+c)^{2}=2(b c+b-c-1)$

$\Leftrightarrow b^{2}+2 b c+c^{2}=2 b c+2 b-2 c-2$

$\Leftrightarrow(b-1)^{2}+(c+1)^{2}=0 \Leftrightarrow\left\{\begin{array}{l}b=1 \\ c=-1 .\end{array}\right.$

Suy ra $a=0$.

Vậy $A=a^{2}+b^{2}+c^{2}=0^{2}+1^{2}+(-1)^{2}=2$.

Bài 2.

a) Giải phương trình: $4 \sqrt{x+3}=1+4 x+\frac{2}{x}$.

b) Giải hệ phương trình: $\left\{\begin{array}{l}x^{2}+y^{3}=1 \\ x^{2}+y^{5}=x^{3}+y^{2}\end{array}\right.$

Lời giải.

a) Giải phương trình: $4 \sqrt{x+3}=1+4 x+\frac{2}{x}$.

Điều kiện xác định: $\left\{\begin{array}{l}x \neq 0 \\ x \geq 3 .\end{array}\right.$

Ta có: $(1) \Leftrightarrow 4 x \sqrt{x+3}=x+4 x^{2}+2 $

$\Leftrightarrow 4 x^{2}-4 x \sqrt{x+3}+(x+3)=1 $

$\Leftrightarrow(2 x-\sqrt{x+3})^{2}=1 $

$\Leftrightarrow\left[\begin{array}{l}2 x-\sqrt{x+3}=1 \\2 x-\sqrt{x+3}=-1\end{array}\right.$

– Trường hợp 1: $2 x-1=\sqrt{x+3} \Leftrightarrow\left\{\begin{array}{l}2 x-1 \geq 0 \\ (2 x-1)^{2}=x+3\end{array} \Leftrightarrow\left\{\begin{array}{l}x \geq \frac{1}{2} \\ 4 x^{2}-5 x-2=0\end{array}\right.\right.$

– Trường hợp 2: $2 x+1=\sqrt{x+3} \Leftrightarrow\left\{\begin{array}{l}2 x+1 \geq 0 \\ (2 x+1)^{2}=x+3\end{array} \Leftrightarrow\left\{\begin{array}{l}x \geq-\frac{1}{2} \\ 4 x^{2}+3 x-2=0\end{array}\right.\right.$

$\Leftrightarrow\left\{\begin{array}{l}x \geq-\frac{1}{2} \\{\left[\begin{array}{l}x=\frac{-3+\sqrt{41}}{8} \\x=\frac{-3-\sqrt{41}}{8}\end{array} \Rightarrow x=\frac{-3+\sqrt{41}}{8}\right.}\end{array}\right.$

Vậy tập nghiệm của phương trình đã cho là $\mathcal{S}=(\frac{5+\sqrt{57}}{8} ; \frac{-3+\sqrt{41}}{8})$

b) $\left\{\begin{array}{l}x^{2}+y^{3}=1 \\x^{2}+y^{5}=x^{3}+y^{2}\end{array}\right.\quad(2)$

Ta có phương trình

$(2) \Leftrightarrow x^{2}(1-x)=y^{2}\left(1-y^{3}\right)=y^{2} x^{2} \Leftrightarrow x^{2}\left(1-x-y^{2}\right)=0 \Leftrightarrow\left[\begin{array}{l}x=0 \\ 1-x-y^{2}=0\end{array}\right.$

– Trường hợp 1: Với $x=0$ thì $(1) \Leftrightarrow y^{3}=1 \Leftrightarrow y=1$.

– Trường hợp 2: Với $1-x-y^{2}=0 \Leftrightarrow x=1-y^{2}$ thì

(1) $\Leftrightarrow\left(1-y^{2}\right)^{2}+y^{3}=1 $

$\Leftrightarrow y^{4}+y^{3}-2 y^{2}=0 $

$\Leftrightarrow y^{2}\left(y^{2}+y-2\right)=0 $

$\Leftrightarrow y^{2}(y-1)(y+2)=0 $

$\Leftrightarrow\left[\begin{array}{l}y=0 \\\\y=1 \\\\y=-2 \end{array}\right. $

  • Với $y=0$ thì $x=1$.

  • Với $y=1$ thì $x=0$.

  • Với $y=-2$ thì $x=-3$.

Vậy hệ phương trình đã cho có nghiệm là: $\mathcal{S}={(0 ; 1),(1 ; 0),(-3 ;-2)}$

Bài 3. Cho tam giác $A B C(A B<A C)$ vuông tại $A$ có đường cao $A H$. Gọi $E, F$ lần lượt là hình chiếu của $H$ lên $A B, A C$.

a) Chứng minh rằng: $B E \sqrt{C H}+C F \sqrt{B H}=A H \sqrt{B C}$

b) Gọi $D$ là điểm đối xứng của $B$ qua $H$ và gọi $O$ là trung điểm của $B C$. Đường thẳng đi qua $D$ và vuông với $B C$ cắt $A C$ tại $K$. Chứng minh rằng: $B K$ vuông góc với $A O$.

Lời Giải.

a) Chứng minh rằng: $B E \sqrt{C H}+C F \sqrt{B H}=A H \sqrt{B C}$.

Ta có: $H E | A C$ (vì cùng vuông góc với $A B$ ).

Suy ra: $\frac{B E}{A B}=\frac{B H}{B C} \quad$ (1) (theo định lí Ta- lét).

Ta lại có: $H F | A B$ (Vi cùng vuông góc với $A C$ ).

Suy ra: $\frac{C F}{A C}=\frac{C H}{B C} \quad$ (2) (theo định lí Ta-lét).

Từ (1) và (2) suy ra  $\frac{B E}{A B}+\frac{C F}{A C}=\frac{B H}{B C}+\frac{C H}{B C}=1 $

$\Rightarrow B E \cdot A C+C F \cdot A B=A B \cdot A C $

$\Leftrightarrow B E \cdot \sqrt{C H \cdot B C}+C F \sqrt{B H \cdot B C}=A H \cdot B C $

Vì $ A H \cdot B C=A B \cdot A C) $

$\Leftrightarrow \sqrt{B C}(B E \sqrt{C H}+C F \sqrt{B H})=\sqrt{B C} \cdot A H \cdot \sqrt{B C} $

$\Leftrightarrow B E \sqrt{C H}+C F \sqrt{B H}=A H \sqrt{B C} . \text { (dpcm) }$

b) Ta có: $\triangle C D K \sim \triangle C A B \quad(g-g)$

Suy ra: $\frac{C D}{C K}=\frac{C A}{C B}$

Xét $\triangle A C D$ và $\triangle B C K$ có:

$C$ chung và $\frac{C D}{C K}=\frac{C A}{C B}(\mathrm{cmt})$.

Suy ra: $\triangle A C D \sim \triangle B C K \quad(c-g-c)$.

Do đó: $\widehat{K B D}=\widehat{C A D}=90^{\circ}-\widehat{B A D}=90^{\circ}-2 \widehat{B A H}=90^{\circ}-2 \widehat{B C A}($ Vi $\widehat{B A H}=$ $\widehat{B C A}$ (cùng phụ với $A B C$) (1)

Mà $\triangle A B C$ vuông tại $A$ có $O$ là trung điểm của $B C$.

Suy ra $O$ là tâm đường tròn ngoại tiếp $\triangle A B C \Rightarrow 2 \widehat{A C B}=\widehat{A O B}$ Khi đó $(1) \Leftrightarrow \widehat{K B D}=90^{\circ}-\widehat{A O B}$.

Vậy $B K \perp A O$ (đpcm).

Bài 4.

a) Chứng minh rằng: $x^{4}-x+\frac{1}{2}>0$ với mọi số thực $x$.

b) Cho $x, y$ là các số thực thỏa mãn điều kiện $x^{2}-x y+y^{2}=3$. Tìm giá trị lớn nhất và giá trị nhỏ nhất của biểu thức $P=x^{2}+y^{2}$.

Lời giải.

a) Ta có: $x^{4}-x+\frac{1}{2}=x^{4}-x^{2}+x^{2}-x+\frac{1}{2}=x^{4}-2 \cdot \frac{1}{2} x^{2}+\frac{1}{4}+x^{2}-2 \cdot \frac{1}{2} x+\frac{1}{4}$ $=\left(x^{2}-\frac{1}{2}\right)^{2}+\left(x-\frac{1}{2}\right)^{2} \geq 0$ với mọi số thực $x$.

Dấu bằng xảy $\mathrm{ra} \Leftrightarrow\left\{\begin{array}{l}x^{2}-\frac{1}{2}=0 \\x-\frac{1}{2}=0\end{array} \Leftrightarrow\left\{\begin{array}{l}x^{2}=\frac{1}{2} \\x=\frac{1}{2}\end{array}\right.\right.$ Hệ phương trình vô nghiệm.

Do đó đẳng thức không xảy ra, tức là $x^{4}-x+12>0$ với mọi số thực $x$.

b) Cách 1:

Ta có: $A=\frac{P}{3}=\frac{x^{2}+y^{2}}{x^{2}-x y+y^{2}}$

Xét $y=0 \Rightarrow x=\pm \sqrt{3} \Rightarrow P=3$.

Xét $y \neq 0$, khi đó $A=\frac{\left(\frac{x}{y}\right)^{2}+1}{\left(\frac{x}{y}\right)^{2}-\frac{x}{y}+1} \quad(1)$. Đặt $t=\frac{x}{y}$.

Khi đó $(1)$ trở thành $A=\frac{t^{2}+1}{t^{2}-t+1} \Leftrightarrow t^{2}(A-1)+A t+A-1=0$.

Xét $A=1 \Rightarrow P=3$.

Xét $A \neq 1 \Rightarrow \Delta=A^{2}-4(A-1)^{2}=-(3 A-2)(A-2) \geq 0 \Leftrightarrow \frac{2}{3} \leq A \leq 2 \Leftrightarrow$ $2 \leq P \leq 6$.

$\operatorname{Min} P=2$ khi $x=1 ; y=-1$ hoặc $x=-1 ; y=1$.

$\operatorname{Max} P=6$ khi $x=y=\pm \sqrt{3}$.

Cách 2: $3=\frac{1}{2}\left(2 x^{2}-2 x y+2 y^{2}\right)=\frac{1}{2}\left[3\left(x^{2}+y^{2}\right)-(x-y)^{2}\right]$.

$\Rightarrow x^{2}+y^{2} \geq 2$

$3=\frac{1}{2}\left(2 x^{2}-2 x y+2 y^{2}\right)=\frac{1}{2}\left[x^{2}+y^{2}+(x-y)^{2}\right] .$

$\Rightarrow x^{2}+y^{2} \leq 6$

Vậy $2 \leq P \leq 6$.

Bài 5. Cho tam giác $A B C$ vuông tại $A$. Gọi $M$ là trung điểm của $B C$ và $O$ là tâm đường tròn ngoại tiếp của tam giác $A M B$. Đường thẳng $A C$ cắt $(O)$ tại điểm thứ hai $K$. Đường thẳng $B K$ cắt đường tròn ngoại tam giác $A B C$ tại $L$. Các đường thẳng $C L$ và $K M$ cắt nhau tại $E$. Chứng minh rằng $E$ nằm trên đường tròn ngoại tiếp tam giác ACM.

Lời giải. Ta có: tứ giác $A K M B$ nội tiếp ( tổng hai góc đối bằng $180^{\circ}$ )

Suy ra: $\widehat{M A K}=\widehat{K B M}=\widehat{L B C}$ (cùng chắn cung $K M$ )

Ta lại có: $\widehat{L B C}=\widehat{M E C}$ (cùng phụ với $\widehat{E C B}$ ) $\Rightarrow \widehat{M A K}=\widehat{M E C}$

Suy ra tứ giác $M A E C$ nội tiếp

Vậy $E$ nằm trên đường tròn ngoại tiếp $\triangle A M C$.

Bài 6. Các số nguyên dương từ 1 đến 2018 được tô màu theo nguyên tắc sau: Các số mà khi chia hết cho 24 dư 17 tô được màu xanh. Các số mà khi chia cho 40 dư 7 được tô màu đỏ. Các số còn lại được tô màu vàng.

a) Chứng tỏ rằng không có số nào được tô cả hai màu xanh và đỏ. Hỏi có bao nhiêu số được tô màu vàng?

b) Có bao nhiêu cặp số $(a, b)$ sao cho $a$ được tô màu xanh, $b$ được tô màu đỏ và $|a-b|$ bằng 2 ?

Lời giải.

a) Theo đề bài ta có

  • Số màu xanh có dạng: $24 x+17 \quad(x \in \mathbb{N})$ với $1 \leq 24 x+17 \leq 2018 \Leftrightarrow-\frac{2}{3} \leq x \leq \frac{667}{8} \Leftrightarrow 0 \leq x \leq 83 .$

Do đó có 84 số được tô màu xanh.

  • Số màu đỏ có dạng: 40y $+7 \quad(y \in \mathbb{N})$ với $1 \leq 40 y+7 \leq 2018 \Leftrightarrow-\frac{3}{20} \leq x \leq \frac{2011}{40} \Leftrightarrow 0 \leq x \leq 50$.

Do đó có 51 số được tô màu đỏ.

  • Giả sử có số được tô cả hai màu xanh và đỏ khi đó tồn tại $x_{0} ; y_{0}$ sao cho $24 x_{0}+17=40_{0} y+7 \Leftrightarrow 24_{0} x+10=40_{0} y$

Vì $24: 4$ và $40: 4$ nên $10: 4$ (vô lí)

Vậy không có số nào được tô cả hai màu xanh và đỏ. Khi đó số lượng số được tô màu vàng là: $2018-84-51=1883$ (số).

b) Ta có $a=24 x+17$ và $b=40 y+7$ với $(x, y \in \mathbb{N})$ Xét 2 trường hợp sau:

  • Trường hợp 1: $a-b=2 \Leftrightarrow 24 x-40 y+10=2 \Leftrightarrow 3 x-5 y=-1 \Leftrightarrow 3 x=$ $5 y-1 .$

Mà $0 \leq x \leq 83 \Leftrightarrow 0 \leq 3 x \leq 249$

Suy ra $0 \leq 5 y-1 \leq 249 \Leftrightarrow \frac{1}{5} \leq y \leq 50$.

Vi $(5 y-1)$ : 3 nên $y$ chia 3 dư 2 .

Kiểm tra ta thấy $y$ nhận 17 giá trị khác nhau $\Rightarrow$ có 17 cặp $(x ; y)$.

  • Trường hợp 2: $a-b=-2 \Leftrightarrow 24 x+17-40 y-7=-2 \Leftrightarrow 24 x-40 y=$ $-12$

$\Leftrightarrow 6 x-10 y=-3$ (loại)

Vậy có 17 cặp $(a ; b)$ thỏa mãn yêu cầu bài toán.

 

 

 

 

 

 

 

 

 

 

 

 

 

 

 

 

 

 

 

 

 

 

 

 

 

 

 

 

 

 

 

 

 

 

 

 

 

 

 

 

ĐỀ THI VÀO LỚP 10 CHUYÊN TOÁN PHỔ THÔNG NĂNG KHIẾU 2018

Bài 1. Cho các phương trình $x^{2}-x+m=0$

(1) và $m x^{2}-x+1=0$

(2) với $m$ là tham số.

(a) Tìm $m$ để các phương trình (1) và (2) đều có 2 nghiệm dương phân biệt.

(b) Giả sử điều kiện ở câu a) được thỏa mãn gọi $x_{1}$; $x_{2}$ là nghiệm của (1) và $x_{3} ; x_{4}$ là nghiệm của (2).

Chứng minh rằng $x_{1} x_{2} x_{3}+x_{2} x_{3} x_{4}+x_{3} x_{4} x_{1}+x_{4} x_{1} x_{2}>5$

Bài 2. Cho $a, b$ là hai số nguyên thỏa mãn $a^{3}+b^{3}>0$.

(a) Chứng minh rằng $a^{3}+b^{3} \geq a+b>0$.

(b) Chứng minh rằng $a^{3}+b^{3} \geq a^{2}+b^{2}$.

(c) Tìm tất cả các bộ số $x, y, z, t$ nguyên sao cho $x^{3}+y^{3}=z^{2}+t^{2}$ và $z^{3}+t^{3}=$ $x^{2}+y^{2}$.

Bài 3. Cho $A_{n}=2018^{n}+2032^{n}-1964^{n}-1984^{n}$ với $n$ là số tự nhiên.

(a) Chứng minh với mọi số tự nhiên $n$ thì $A_{n}$ chia hết cho 51 .

(b) Tìm tất cả những số tự nhiên $n$ sao cho $A_{n}$ chia hết cho 45 .

Bài 4. Cho tam giác $A B C$ nhọn. Một đường tròn qua $B, C$ cắt các cạnh $A B, A C$ lần lượt tại $E$ và $F ; B F$ cắt $C E$ tại $D$. Lây điểm $K$ sao cho từ giác $D B K C$ là hình bình hành.

(a) Chứng minh rằng $\triangle K B C$ đồng dạng với $\triangle D F E, \triangle A K C$ dồng dạng với $\triangle A D E$.

(b) Hạ $D M$ vuông góc với $A B, D N$ vuông góc với $A C$. Chứng minh rằng $M N$ vuông góc với $A K$.

(c) Gọi $I$ là trung điểm $A D$, $J$ là trung điểm $M N$. Chứng minh rằng đường thẳng $I J$ đi qua trung điểm của cạnh $B C$.

(d) Đường thẳng $I J$ cắt đường tròn ngoại tiếp tam giác $I M N$ tại $T(T \neq I)$. Chứng minh rằng $A D$ tiếp xúc với đường tròn ngoại tiếp tam giác $D T J$.

Bài 5. Đội văn nghệ của một trường THCS có 8 học sinh. Nhà trường muốn thành lập các nhóm tốp ca, mỗi nhóm gồm đúng 3 học sinh, (mỗi học sinh có thể tham gia vài nhóm tốp ca khác nhau). Biết rằng hai nhóm tốp ca bất kỳ có chung nhau nhiều nhất là một học sinh.

(a) Chứng minh rằng không có học sinh nào tham gia từ 4 nhóm tốp ca trở lên.

(b) Có thể thành lập nhiều nhất là bao nhiêu nhóm tốp ca như vậy?

LỜI GIẢI

 

Bài 1. Cho các phương trình $x^{2}-x+m=0 \quad$ (1) và $m x^{2}-x+1=0$

(2) với $m$ là tham số.

(a) Tìm $m$ để các phương trình (1) và $(2)$ đều có 2 nghiệm dương phân biệt.

(b) Giả sử điều kiện ở câu a) được thỏa mãn gọi $x_{1}$; $x_{2}$ là nghiệm của (1) và $x_{3} ; x_{4}$ là nghiệm của $(2)$.

Chứng minh rằng $x_{1} x_{2} x_{3}+x_{2} x_{3} x_{4}+x_{3} x_{4} x_{1}+x_{4} x_{1} x_{2}>5$

Lời giải.

(a) Xét phương trình (1): $x^{2}-x+m=0$

Phương trình (1) có hai nghiệm dương phân biệt:

$\left\{\begin{array}{l}\Delta>0 \\ S>0 \ P>0\end{array} \Leftrightarrow\left\{\begin{array}{l}1-4 m>0 \\ 1>0 \ m>0\end{array} \Leftrightarrow\left\{\begin{array}{l}m<\frac{1}{4} \\m>0\end{array} \Leftrightarrow 0<m<\frac{1}{4}\right.\right.\right.$

Phương trình (2) có hai nghiệm dương phân biệt:

$\left\{\begin{array}{l}m \neq 0 \\ \Delta>0 \\ S>0 \\ P>0\end{array} \Leftrightarrow\left\{\begin{array}{l}m \neq 0 \\ 1-4 m>0 \\ \frac{1}{m}>0 \\ \frac{1}{m}>0\end{array} \Leftrightarrow\left\{\begin{array}{l}m \neq 0 \\ m<\frac{1}{4} \\ m>0\end{array} \Leftrightarrow 0<m<\frac{1}{4}\right.\right.\right.$

Vậy để $(1)$ và $(2)$ có hai nghiệm dương phân biệt thì $0<m<\frac{1}{4}$

b) Theo Viet ta có: $\left\{\begin{array}{l}x_{1}+x_{2}=1 \\ x_{1} x_{2}=m \\ x_{3}+x_{4}=\frac{1}{m} \\ x_{3} x_{4}=\frac{1}{m}\end{array}\right.$

$\text { Ta có } x_{1} x_{2} x_{3}+x_{2} x_{3} x_{4}+x_{3} x_{4} x_{1}+x_{4} x_{1} x_{2}$

$=x_{1} x_{3}+\frac{x_{2}}{m}+\frac{x_{1}}{m}+m x_{4}$

$=m\left(x_{3}+x_{4}\right)+\frac{1}{m}\left(x_{1}+x_{2}\right)$

$=1+\frac{1}{m}>1+\frac{1}{\frac{1}{4}}=5(\text { dpcm }) .$

Bài 2. Cho $a, b$ là hai số nguyên thỏa mãn $a^{3}+b^{3}>0$.

(a) Chứng minh rằng $a^{3}+b^{3} \geq a+b>0$.

(b) Chứng minh rằng $a^{3}+b^{3} \geq a^{2}+b^{2}$.

(c) Tìm tất cả các bộ số $x, y, z, t$ nguyên sao cho $x^{3}+y^{3}=z^{2}+t^{2}$ và $z^{3}+t^{3}=x^{2}+y^{2} .$

Lời giải. $a, b \in \mathbb{Z}: a^{3}+b^{3}>0$

(a) $a^{3}+b^{3}>0 \Leftrightarrow(a+b)\left(a^{2}-a b+b^{2}\right)>0$

Do $a^{2}-a b+b^{2}=\left(a-\frac{b}{2}\right)^{2}+\frac{3 b^{2}}{4} \geq 0$. Dấu “=” xảy ra $\Leftrightarrow a=b=0$ (loại).

$\Rightarrow a^{2}-a b+b^{2}>0$ nên $a+b>0$ (đpcm).

Ta có: $a^{3}+b^{3} \geq a+b$

$\Leftrightarrow(a+b)\left(a^{2}-a b+b^{2}-1\right) \geq 0 \quad (* *)$

Do $\left\{\begin{array}{l}a^{2}-a b+b^{2}>0 \\ a, b \in \mathbb{Z}\end{array} \Rightarrow a^{2}-a b+b^{2} \geq 1\right.$ nên $(* *)$ đúng.

Vậy $a^{3}+b^{3} \geq a+b$ và dấu “=” xảy ra $\Leftrightarrow\left\{\begin{array}{l}a=1 \\ b=0\end{array}\right.$ hay $\left\{\begin{array}{l}a=0 \\ b=1\end{array}\right.$ hay $\left\{\begin{array}{l}a=1 \\ b=1\end{array}\right.$

(b) Cách 1:

Do $a+b>0 \Rightarrow a+b \geq 1$.

TH1: $a+b=1 \Leftrightarrow b=1-a$.

Ta có: $a^{3}+b^{3} \geq a^{2}+b^{2} \Leftrightarrow a^{3}+(1-a)^{3} \geq a^{2}+(1-a)^{2}$

$\Leftrightarrow a^{2}-a \geq 0$

$\Leftrightarrow a \leq 0$ hoặc $a \geq 1$ (đúng vì $a \in \mathbb{Z}$ )

Vậy $a^{3}+b^{3} \geq a^{2}+b^{2}$ và dấu “=” xảy ra $\Leftrightarrow(a ; b) \in{(0 ; 0) ;(1 ; 1) ;(0 ; 1) ;(1 ; 0)}$.

TH2: $a+b \geq 2$

Ta có: $a^{3}+b^{3}=(a+b)\left(a^{2}-a b+b^{2}\right) \geq 2\left(a^{2}-a b+b^{2}\right)=a^{2}+b^{2}+$ $(a-b)^{2} \geq a^{2}+b^{2}$.

Cách 2:

Rõ ràng $a, b$ không thể đồng thời bé hơn 0 .

TH1: $a=b=0$ : hiển nhiên $a^{3}+b^{3} \geq a^{2}+b^{2}$

TH2: Một trong hai số bằng 0 , số còn lại khác 0 .

Giả sử: $\left\{\begin{array}{l}b=0 \\ a \neq 0\end{array} \Rightarrow a>1 \Rightarrow a^{3} \geq a^{2} \Rightarrow a^{3}+b^{3} \geq a^{2}+b^{2}\right.$

Dấu “=” xảy ra khi $a=1$.

TH3: $a, b \geq 1 \Rightarrow\left\{\begin{array}{l}a^{3} \geq a^{2} \\ b^{3} \geq b^{2}\end{array} \Rightarrow a^{3}+b^{3} \geq a^{2}+b^{2}\right.$

TH4: $\left\{\begin{array}{l}a>0 \\ b<0\end{array} \Rightarrow\left\{\begin{array}{l}a \geq 1 \\ b \leq-1\end{array}\right.\right.$

Đặt $a=|b|+k, k>1$

$a^{3}+b^{3} \geq a^{2}+b^{2}$

$\Leftrightarrow(|b|+k)^{3}+b^{3} \geq(|b|+k)^{2}+b^{2}$

$\Leftrightarrow 3|b|^{2} k+3|b| k^{2}+k^{3} \geq 2|b|^{2}+2|b| k+k^{2}$

$\left.\Rightarrow 3 b^{2} k+3|b| k+k^{3} \geq 2 b^{2}+2|b| k+k^{2} \quad \text { (Do k }>1\right)$

$\Leftrightarrow(3 k-2) b^{2}+|b| k+k^{2}(k-1) \geq 0 \text { (đúng). }$

Vậy $a^{3}+b^{3} \geq a^{2}+b^{2}$.

(c) Từ giả thiết $\Rightarrow x^{3}+y^{3} \geq 0 ; z^{3}+t^{3} \geq 0$.

Nếu $x^{3}+y^{3}=0 \Rightarrow z^{2}+t^{2}=0 \Rightarrow z=t=0$

$\Rightarrow x^{2}+y^{2}=0 \Rightarrow x=y=0 \text {. }$

Nếu $z^{3}+t^{3}=0$, tương tự ta có $x=y=z=t=0$.

Nếu $\left\{\begin{array}{l}x^{3}+y^{3}>0 \\ z^{3}+t^{3}>0\end{array}\right.$

Từ giả thiết suy ra $\left(x^{3}+y^{3}\right)+\left(z^{3}+t^{3}\right)=x^{2}+y^{2}+z^{2}+t^{2}(* * *)$

Theo câu b) : $\left\{\begin{array}{l}x^{3}+y^{3} \geq x^{2}+y^{2} \\ z^{3}+t^{3} \geq z^{t}+t^{2}\end{array}\right.$

Nếu $(* * *) \Leftrightarrow(x ; y),(z, t)$ là một trong các bộ $(1 ; 1) ;(1 ; 0) ;(0 ; 1)$.

Vậy nghiệm phương trình:

$(x, y, z, t) \in{(0 ; 0 ; 0 ; 0),(1 ; 1 ; 1 ; 1),(1 ; 0 ; 0 ; 1),(0 ; 1 ; 1 ; 0),(1 ; 0 ; 1 ; 0),(0 ; 1 ; 0 ; 1)} \text {. }$

Bài 3. Cho $A_{n}=2018^{n}+2032^{n}-1964^{n}-1984^{n}$ với $n$ là số tự nhiên.

(a) Chứng minh với mọi số tự nhiên $n$ thì $A_{n}$ chia hết cho 51 .

(b) Tìm tất cả những số tự nhiên $n$ sao cho $A_{n}$ chia hết cho 45 .

Lời giải.

(a) Do $2018 \equiv 1964 \quad(\bmod 3) \Rightarrow 2018^{n} \equiv 1964^{n} \quad(\bmod 3)$. $2032 \equiv 1984 \quad(\bmod 3) \Rightarrow 2032^{n} \equiv 1984^{n} \quad(\bmod 3) .$

$\Rightarrow A_{n} \vdots 3 .$

Ta lại có $2018 \equiv 1984 \quad(\bmod 17) \Rightarrow 2018^{n} \equiv 1984^{n} \quad(\bmod 17)$. $2032 \equiv 1964 \quad(\bmod 17) \Rightarrow 2032^{n} \equiv 1964^{n} \quad(\bmod 17) .$ $\Rightarrow A_{n} \vdots 17 .$

Do $(3 ; 17)=1$ nên $A_{n}: 51 \quad \forall n$

(b) $A_{n}=2018^{n}+2032^{n}-1964^{n}-1984^{n}$.

  • Ta xét các trường hợp của $n$ để $A_{n} \vdots 5$.

Ta có $A_{n} \equiv(-2)^{n}+2^{n}-2 \cdot(-1)^{n}(\bmod 5)$.

Do đó nếu $n$ lẻ $\Rightarrow A_{n} \equiv 2 \quad(\bmod 5) \quad$ (loại).

Nếu $n=4 k \Rightarrow A_{n} \equiv 2 \cdot 2^{4 k}-2 \equiv 2-2 \equiv 0 \quad(\bmod 5)$ (nhận)

Nếu $n=4 k+2 \Rightarrow A_{n} \equiv 2 \cdot 2^{4 k+2}-2 \equiv 8-2 \equiv 6(\bmod 5)$ (loại). Vậy $A_{n} \vdots 5 \Leftrightarrow n \vdots 4$.

  • Ta xét các trường hợp của $n$ để $A_{n}: 9$.

Ta có

$\begin{aligned} A_{n} & \equiv 2^{n}+(-2)^{n}-2^{n}-4^{n} \quad(\bmod 9) \\ & \equiv 2^{n}-4^{n} \quad(\bmod 9) \quad(\text { Do n chẵn }) \\ & \equiv 2^{n}\left(1-2^{n}\right) \quad(\bmod 9) \end{aligned}$

$\operatorname{Vi}(2 ; 9)=1 \Rightarrow 2^{n}-1: 9 .$

Xét $n=3 k$ với $k \in \mathbb{N}$. Ta có $A_{n} \equiv 2^{3 k}-1 \equiv(-1)^{k}-1 \quad(\bmod 9) \Rightarrow k$ chẵn

Xét $n=3 k+1$ với $k \in \mathbb{N}$. Ta có $A_{n} \equiv 2^{3 k+1}-1 \equiv 2 \cdot(-1)^{k}-$ $1(\bmod 9)$ (loại).

Xét $n=3 k+2$ với $k \in \mathbb{N}$. Ta có $A_{n} \equiv 2^{3 k+2}-1 \equiv 4 \cdot(-1)^{k}-$ $1(\bmod 9)$ (loại).

Vậy $A_{n} \vdots 45 \Leftrightarrow n \vdots 12$.

Bài 4. Cho tam giác $A B C$ nhọn. Một đường tròn qua $B, C$ cắt các cạnh $A B, A C$ lần lượt tại $E$ và $F ; B F$ cắt $C E$ tại $D$. Lấy điểm $K$ sao cho từ giác $D B K C$ là hình bình hành.

(a) Chứng minh rằng $\triangle K B C$ đồng dạng với $\triangle D F E, \triangle A K C$ đồng dạng với $\triangle A D E$.

(b) Hạ $D M$ vuông góc với $A B, D N$ vuông góc với $A C$. Chứng minh rằng

$M N$ vuông góc với $A K$.

(c) Gọi $I$ là trung điểm $A D, J$ là trung điểm $M N$. Chứng minh rằng đường thẳng $I J$ đi qua trung điểm của cạnh $B C$.

(d) Đường thẳng $I J$ cắt đường tròn ngoại tiếp tam giác $I M N$ tại $T(T \neq I)$. Chứng minh rằng $A D$ tiếp xúc với đường tròn ngoại tiếp tam giác $D T J$.

Lời giải.

(a) Tứ giác $B E F C$ nội tiếp nên $\angle D E F=\angle D B C$ và $\angle D F E=\angle D C B$.

Và $B D C K$ là hình bình hành nên $\angle D B C=\angle K C B, \angle D C B=\angle K B C$

Do đó $\angle D E F=\angle K C B, \angle D F E=\angle K B C$, suy ra $\triangle K B C \sim \triangle D F E$

Ta có $\angle A E C=\angle A B K$ và $\angle A B K=\angle A B D+\angle D B K=\angle A C E+\angle D C K=$ $\angle A C K$ (do $\angle A B D=\angle A C E, \angle D B K=\angle D C K)$

Do $\triangle D E F \sim \triangle K C B$ nên $\frac{D E}{C K}=\frac{E F}{B C}$ (1)

Mặt khác $\triangle A E F \sim \triangle A C B$ nên $\frac{E F}{B C}=\frac{A E}{A C}$ (2)

Từ (1) và (2) suy ra $\frac{D E}{C K}=\frac{A E}{A C}$

Xét $\triangle A E D$ và $\triangle A C K$ có $\angle A E D=\angle A C K, \frac{D E}{C K}=\frac{A E}{A C}$

Suy ra $\triangle A E D \sim \triangle A C K(\mathrm{c}-\mathrm{g}-\mathrm{c})$

(b) Gọi $Q$ là giao điểm của $A K$ và $M N$

Ta có $\triangle D A E \backsim \triangle K A C$ nên $\angle K A C=\angle D A E$ hay $\angle Q A C=\angle D A M$

Tứ giác $A M D N$ có $\angle A M D+\angle A N D=90^{\circ}+90^{\circ}=180^{\circ}$ nên nội tiếp.

Suy ra $\angle D N M=\angle D A M=\angle Q A N$

Mà $\angle D N M+\angle M N A=90^{\circ}$, suy ra $\angle Q A N+\angle M N A=90^{\circ}$

Suy ra $\angle A Q N=90^{\circ}$. Vậy $A K \perp M N$.

(c) Cách 1. Ta có $I J \perp M N$ và $A K \perp M N$, suy ra $I J | A K$.

Mà $I$ là trung điểm $A D$, suy ra $I J$ qua trung điểm $P$ của $D K$. Lại có $D B K C$ là hình bình hành nên $P$ cũng là trung điểm $B C$.

Cách 2. Gọi $P$ là trung điểm của $B C$. $V, U$ lần lượt là trung điểm của $D B, D C$.

Ta có $M I=\frac{1}{2} A D=N I$, suy ra $I$ thuộc trung trực của $M N$.

Ta có $M V=\frac{1}{2} B D\left(\triangle D B M\right.$ vuông tại $M$ ) và $P U=\frac{1}{2} D B$ (đường trung bình)

Suy ra $M V=P U$

Tương tự thì ta có $P V=N U$

Ta có: $\angle M V D=2 \angle M B D=2 \angle N C D=\angle N U D$ và $\angle D V P=\angle D U P$

Suy ra $\angle M V P=\angle P U N$

Xét $\triangle M V P$ và $\triangle P U N$ có $M V=P U, P V=N U, \angle M V P=\angle P U N$

$\Rightarrow \triangle M V P=\triangle P U N(\mathrm{c}-\mathrm{g}-\mathrm{c})$

Suy ra $P M=P N$. Do đó $P$ thuộc trung trực của $M N$.

Vậy $I, P, J$ thuộc trung trực $M N$ nên $I, P, J$ thẳng hàng hay $I J$ qua trung điểm $P$ của $B C$.

(d) Ta có tam giác $I M N$ cân tại $I, I J \perp M N$ nên $I T$ là đường kính của đường tròn ngoại tiếp $\triangle I M N$

Suy ra $\angle I N T=90^{\circ}$.

Suy ra $I J \cdot I T=I N^{2}$ mà $I N=I D$ suy ra $I J \cdot I T=I D^{2}$

Do đó $I D^{2}=I J \cdot I T$. Suy ra $\triangle I D J \sim \triangle I T D(\mathrm{c}-\mathrm{g}-\mathrm{c})$ nên $\angle I D J=\angle I T D$

Từ đó ta có $I D$ là tiếp tuyến của đường tròn ngoại tiếp $\triangle D T J$.

Bài 5. Đội văn nghệ của một trường THCS có 8 học sinh. Nhà trường muốn thành lập các nhóm tốp ca, mỗi nhóm gồm đúng 3 học sinh, (mỗi học sinh có thể tham gia vài nhóm tốp ca khác nhau). Biết rằng hai nhóm tốp ca bất

kỳ có chung nhau nhiều nhất là một học sinh.

(a) Chứng minh rằng không có học sinh nào tham gia từ 4 nhóm tốp ca trở lên.

(b) Có thể thành lập nhiều nhất là bao nhiêu nhóm tốp ca như vậy?

Lời giải.

(a) Giả sử có 1 học sinh tham gia 4 nhóm $A, B, C, D$ là $x$.

Khi đó $A={(x, a, b)} \quad B={(x, c, d)} \quad C={(x, e, f)} \quad D={(x, g, h)}$.

Vi các nhóm không có chung quá 1 thành viên nên các học sinh: $a, b, c, d, e, f, g, h$

là khác nhau (vô lí vì chỉ có 8 học sinh tham gia).

(b) Ta chứng minh lập được nhiều nhất là 8 nhóm.

Thật vậy, nếu có 9 nhóm, mối nhóm có 3 học sinh thì khi đó số lượt học sinh tham gia là $9 \cdot 3=27$ lượt tham gia.

Mà chỉ có 8 học sinh nên theo nguyên lý Dirichlet thì có ít nhất một học sinh có nhiều hơn hoặc bằng 4 lượt (mâu thuẫn do câu $a$ ).

(Một học sinh tham gia 1 nhóm tính là 1 lượt).

Gọi 8 học sinh là $a, b, c, d, e, f, g, h$.

8 nhóm học sinh được chia như sau:

${(a, b, c)} ; \quad{(h, b, e)} ; \quad{(b, d, f)} ; \quad{(a, d, e)} ;$

${(h, c, f)} ;  \quad{(c, e, g)} ; \quad{(a, f, g)} ; \quad{(h, d, g)} .$

 

 

 

 

 

 

 

 

 

 

 

 

 

 

 

 

 

 

 

 

 

 

 

 

 

 

 

 

 

 

 

 

 

 

 

 

 

 

 

 

 

 

 

 

 

 

 

 

 

 

 

 

 

 

 

 

 

Đề thi và đáp án tuyển sinh vào 10 TPHCM 2018

Đề thi vào lớp 10 TPHCM năm 2018

 

Bài 1.  Cho parabol $(P):y=x^2$ và đường thẳng $(d):y=3x-2$.

a) Vẽ $(P)$ và $(d)$ trên cùng hệ trục tọa độ.

b) Tìm tọa độ giao điểm của $(P)$ và $(d)$ bằng phép tính.

Giải

a) Học sinh tự vẽ hình.

b) Phương trình hoành độ giao điểm của $(P)$ và $(d)$ là:

$x^2=3x-2 \Leftrightarrow x^2 -3x+2 =0 \Leftrightarrow (x-1)(x-2)=0 \Leftrightarrow \left[ \begin{array}{l} x=1 \\ x=2 \end{array} \right. $

  • Với $x=1$, suy ra $y=1$
  • Với $x=2$, suy ra $y=4$

Vậy giao điểm của $(P)$ và $(d)$ là $(1;1)$ và $(2;4)$

Bài 2. Cho phương trình: $3x^2-x-1=0$ có hai nghiệm $x_1$, $x_2$.

Không giải phương trình, hãy tính giá trị của biểu thức $A=x_1^2+x_2^2$.

Giải

$3x^2-x-1=0$

Ta có: $\Delta = 1-4.3.(-1)=13>0$ nên phương trình trên luôn có hai nghiệm $x_1$, $x_2$.

Theo định lý Viete, ta có: $\left\{ \begin{array}{l} S=x_1+x_2=-\dfrac{b}{a}=\dfrac{1}{3} \\ P=x_1.x_2= \dfrac{c}{a}= -\dfrac{1}{3} \end{array} \right. $

$A=x_1^2 + x_2^2 = \left( x_1 +x_2 \right) ^2 -2x_1x_2 = \left( \dfrac{1}{3} \right) ^2 -2. \dfrac{-1}{3} = \dfrac{7}{9}$

Bài 3. Mối quan hệ giữa thang đo nhiệt độ $F$ (Fahrenheit) và thang đo nhiệt độ $C$ (Celsius) được cho bởi công thức $T_F=1,8T_C +32$, trong đó $T_C$ là nhiệt độ tính theo độ $C$ và $T_F$ là nhiệt độ tính theo độ $F$.

Ví dụ: $T_C= 0^\circ C$ tương ứng với $T_F=32^\circ F$.

a) Hỏi $25^\circ C$ ứng với bao nhiêu độ $F$?

b) Các nhà khoa học đã tìm ra mối liên hệ giữa $A$ là số tiếng kêu của một con dế trong một phút và $T_F$ là nhiệt độ cơ thể của nó bởi công thức: $A=5,6.T_F-275$, trong đó nhiệt độ $T_F$ tính theo độ $F$. Hỏi nếu con dế kêu $106$ tiếng trong một phút thì nhiệt độ của nó khoảng bao nhiêu độ $C$? (làm tròn đến hàng đơn vị)

Giải

a) Với $T_C= 25^\circ C$ thì: $T_F=1,8.25+32=77 \left( ^\circ F \right) $

b) Nếu con dế kêu 106 tiếng trong một phút thì ta có:

$106=5,6.T_F-275 \Leftrightarrow T_F=\dfrac{1905}{28} \left( ^\circ F \right) $

Nhiệt độ con dế tính theo độ $C$:

$T_F = 1,8. T_C +32 \Leftrightarrow \dfrac{1905}{28}=1,8 .T_C +32 \Leftrightarrow T_C \approx 20,02 \left( ^\circ C \right) $

Bài 4. Kim tự tháp Kheops – Ai Cập có dạng hình chóp đều, đáy là hình vuông, các mặt bên là tam giác cân chung đỉnh (hình vẽ). Mỗi cạnh bên của kim tự tháp dài $214 \; m$, cạnh đáy của nó dài $230 \; m$.

a) Tính theo mét chiều cao $h$ của kim tự tháp (làm tròn đến chữ số thập phân thứ nhất).

b) Cho biết thể tích của hình chóp được tính theo công thức $V=\dfrac{1}{3}S.h$, trong đó $S$ là diện tích mặt đáy, $h$ là chiều cao của hình chóp. Tính theo $m^3$ thể tích của kim tự tháp này (làm tròn đến hàng nghìn).

Giải

a) Xét $\triangle BCD$ vuông tại $C$, ta có:

$BD^2 = BC^2 + CD^2$

$\Leftrightarrow BD^2 = 230^2 + 230^2 $

$\Leftrightarrow BD = 230\sqrt{2} \; (m)$ $

$\Rightarrow DO = \dfrac{BD}{2}= 115\sqrt{2} \; m$

$\triangle  SOD$ vuông tại $O$ có:

$SO^2 + OD^2 = SD^2 $

$\Leftrightarrow h^2 + \left( 115\sqrt{2} \right) ^2 = 214^2 $

$\Leftrightarrow h^2 = 19346 \Leftrightarrow h \approx 139,1 \; m$

Vậy $h \approx 139,1 \; m$

b) $S_{ABCD} = BC^2 = 230^2 \; \left( m^2 \right) $

Suy ra: $V_{ABCD} = \dfrac{1}{3}. S_{ABCD}.h= \dfrac{1}{3}. 230^2 .\sqrt{19346} \approx 2453000 \; \left( m^3 \right) $

Bài 5. Siêu thị $A$ thực hiện chương trình giảm giá cho khách hàng mua loại túi bột giặt $4kg$ như sau: Nếu mua $1$ túi thi được giảm $10 000$ đồng so với giá bán niêm yết. Nếu mua $2$ túi thì túi thứ nhất được giảm $10 000$ đồng và túi thứ hai được giảm $20 000$ đồng so với giá niêm yết. Nếu mua từ $3$ túi trở lên thì ngoài $2$ túi đầu được hưởng như chương trình giảm giá như trên, từ túi thứ ba trở đi, mỗi túi sẽ được giảm $20\%$ so với giá niêm yết.

a) Bà Tư mua $5$ túi bột giặt loại $4kg$ ở siêu thị $A$ thì phải trả số tiền là bao nhiêu, biết rằng loại túi bột giặt bà Tư mua có giá niêm yết là $150 000$ đồng/túi.

b) Siêu thị $B$ lại có hình thức giảm giá khác cho loại túi bột giặt nêu trên là: nếu mua từ $3$ túi trở lên thì sẽ giảm giá $15\%$ cho mỗi túi. Nếu bà Tư mua $5$ túi bột giặt thì bà Tư nên mua ở siêu thị nào để số tiền phải trả là ít hơn? Biết rằng giá niêm yết của hai siêu thị là như nhau.

Giải

a) Giá bà Tư phải trả cho túi thứ nhất:

$$ 150 000-10000=140000 \text{ (đồng)} $$

Giá bà Tư phải trả cho túi thứ hai:

$$ 150 000-20000=130000 \text{ (đồng)} $$

Giá bà Tư phải trả cho từ túi thứ 3 đến túi thứ 5:

$$3. 150 000. \left( 100\% -20\% \right) =360000 \text{ (đồng)} $$

Tổng số tiền bà Tư phải trả ở siêu thị $A$:

$$ 140000+130000+360000=630000 \text{ (đồng)}$$

b) Số tiền bà Tư phải trả khi mua 5 túi ở siêu thị $B$:

$$5.150000.\left( 100\% -15\% \right) = 637500 \text{ (đồng)}$$.

Vậy bà Tư nên mua ở siêu thị $A$.

Bài 6. Nhiệt độ sôi của nước không phải lúc nào cũng là $100^\circ C$ mà phụ thuộc vào độ cao của nơi đó so với mực nước biển. Chẳng hạn, Thành phố Hồ Chí Minh có độ cao xem như ngang mực nước biển ($x=0m$) thì nước sôi ở nhiệt độ là $y=100^\circ C$, nhưng ở thủ đô La Paz của Bolivia, Nam Mỹ có độ cao $x=3600m$ so với mực nước biển thì nhiệt độ sôi của nước là $y=87^\circ C$. Ở độ cao trong khoảng vài $km$, ngườu ta thấy mối liên hệ giữa hai đại lượng này là một hàm số bậc nhất $y=ax+b$ có đồ thị như sau:

trong đó $x$ là đại lượng biểu thị cho độ cao so với mực nước biển, $y$ là đại lượng biểu thị cho nhiệt độ sôi của nước.

a) Xác định các hệ số $a$ và $b$.

b) Thành phố Đà Lạt có độ cao $1500m$ so với mực nước biển. Hỏi nhiệt độ sôi của nước ở thành phố này là bao nhiêu?

Giải

a) Ta có: $y=ax+b$ $(1)$.

Dựa vào đồ thị, ta có:

  • Với $x=0$ thì $y=100$, thay vào $(1)$, ta có:

$100=a.0+b \Leftrightarrow b=100$

Suy ra hàm số: $y=ax+100$ $(2)$

  • Với $x=3600$ thì $y=87$, thay vào $(2)$, ta có:

$87=a.3600+100 \Leftrightarrow a=\dfrac{-13}{3600}$

Vậy mối liên hệ là hàm số: $y=-\dfrac{13}{3600}x+100$, hay $a=-\dfrac{13}{3600}$ và $b=100$

b) Nhiệt độ sôi ở Đà Lạt ($x=1500$) là:

$y=-\dfrac{13}{3600}.1500+100 \approx 94,6 \; \left( ^\circ C \right) $

Bài 7. Năm học $2017-2018$, Trường THCS Tiến Thành có ba lớp $9$ gồm $9A$, $9B$, $9C$ trong đó lớp $9A$ có $35$ học sinh và lớp $9B$ có $40$ học sinh. Tổng kết cuối năm học, lớp $9A$ có $15$ học sinh đạt danh hiệu học sinh giỏi, lớp $9B$ có $12$ học sinh đạt danh hiệu học sinh giỏi, lớp $9C$ có $20\%$ đạt danh hiệu học sinh giỏi và toàn khối $9$ có $30\%$ đạt danh hiệu học sinh giỏi. Hỏi lớp $9C$ có bao nhiêu học sinh?

Giải

Gọi $x$ (học sinh) là số học inh của lớp $9C$. ($x\in \mathbb{N}^*$)

Tổng số học sinh giỏi của khối $9$ là: $15+12+x.20\% = 27 + \dfrac{x}{5}$ (học sinh)

Tổng số học sinh của khối $9$: $35+40+x=75+x$ (học sinh)

Ta có:  $\dfrac{27+\dfrac{x}{5}}{75+x}=30\% $

$\Leftrightarrow 27 + \dfrac{x}{5} = \dfrac{3}{10} \left( 75+x \right) $

$\Leftrightarrow 27 + \dfrac{x}{5}= \dfrac{45}{2}+ \dfrac{3}{10}x $

$\Leftrightarrow \dfrac{1}{10}x= \dfrac{9}{2} $

$\Leftrightarrow x=45$ (nhận)

Vậy lớp $9C$ có $45$ học sinh.

Bài 8. Cho tam giác $ABC$ có $BC=8cm$. Đường tròn tâm $O$ đường kính $BC$ cắt $AB$, $AC$ lần lượt tại $E$ và $D$. Hai đường thẳng $BD$ và $CE$ cắt nhau tại $H$.

a) Chứng minh: $AH$ vuông góc với $BC$.

b) Gọi $K$ là trung điểm của $AH$. Chứng minh tứ giác $OEKD$ nội tiếp.

c) Cho $\angle BAC = 60^\circ $. Tính độ dài đoạn $DE$ và tỉ số diện tích hai tam giác $AED$ và $ABC$.

Giải

a) $\triangle ABC$ có:  $\left. \begin{array}{l} CH\bot AB \\ BH \bot AC \end{array} \right\} \Rightarrow H$ là trực tâm của $\triangle ABC \Rightarrow  $AH \bot BC$.

b) $\triangle AEH$ và $\triangle ADH$ lần lượt vuông tại $E$ và $D$

Nên $4$ điểm $A, E, H, D$ cùng nằm trên đường tròn đường kính $AH$ hay đường tròn tâm $K$.

$\Rightarrow \angle BAC = \dfrac{1}{2} \angle EKD$.

Lại có $\angle ABD = \dfrac{1}{2} \angle DOE$ nên

$\angle BAC + \angle ABD = \dfrac{1}{2} \left( \angle EKD + \angle DOE \right) $

$\Rightarrow 180^\circ – ADB = \dfrac{1}{2} \left( \angle EKD + \angle DOE \right) $

$\Rightarrow 90^\circ = \dfrac{1}{2} \left( \angle EKD + \angle DOE \right) $

$\Rightarrow \angle EKD + \angle DOE = 180^\circ $

Vậy $KDOE$ nội tiếp.

c) $\angle A =60^\circ \Rightarrow \angle EKD = 120^\circ \Rightarrow \angle DOE = 60^\circ$

$\triangle DOE$ cân tại $O$ có $\angle DOE =60^\circ $ nên $\triangle DOE$ đều.

$\Rightarrow DE=DO=EO=4cm$

Lại có $\triangle ADE \backsim \triangle ABC$ $(g-g)$ nên

$\dfrac{S_{ADE}}{S_{ABC}}= \left( \dfrac{AD}{AB} \right) ^2 = \left( \cos \angle BAC \right) ^2 = \left( \cos 60^\circ \right) ^2 = \dfrac{1}{4}$

Đề thi và đáp án tuyển sinh vào lớp 10 PTNK không chuyên 2018

Đề thi vào lớp 10 trường Phổ thông Năng khiếu năm 2018

Bài 1. Biết $0<x\le y$ và

$$ \left( \dfrac{\left( \sqrt{x} + \sqrt{y} \right) ^2+\left( \sqrt{x}-\sqrt{y} \right) ^2 }{\left( \sqrt{x} + \sqrt{y} \right) \left( \sqrt{x} -\sqrt{y} \right) + 2\left( x+2y \right) } \right) + \left( \dfrac{y}{\sqrt{x} \left( \sqrt{x}+ \sqrt{y} \right) } + \dfrac{x}{\sqrt{y} \left( \sqrt{x}+\sqrt{y} \right) }\right) = \dfrac{5}{3} $$

Tính $\dfrac{x}{y}$

Giải

Với $0 < x \le y$, ta có:

$\left(\dfrac{(\sqrt x + \sqrt y)^2 + (\sqrt x – \sqrt y)^2}{(\sqrt x + \sqrt y)(\sqrt x – \sqrt y)+2(x+ 2y)}\right) + \dfrac{y}{\sqrt x (\sqrt x + \sqrt y)}+ \dfrac{x}{\sqrt y (\sqrt x + \sqrt y)}= \dfrac{5}{3}$

$\Leftrightarrow \left(\dfrac{2(x+y)}{x-y + 2x + 4y}\right) + \dfrac{y\sqrt y + x\sqrt x}{\sqrt {xy}(\sqrt x + \sqrt y)} = \dfrac{5}{3}$

$\Leftrightarrow \dfrac{2(x+y)}{3(x+y)} + \dfrac{ (\sqrt x + \sqrt y)(x- \sqrt{xy} + y)}{\sqrt{xy}{(\sqrt x+ \sqrt y)}} = \dfrac{5}{3}$

$\Leftrightarrow \dfrac{2}{3} + \dfrac{x – \sqrt {xy} + y}{\sqrt{xy}} = \dfrac{5}{3}$

$\Leftrightarrow \dfrac{x- \sqrt{xy} + y}{\sqrt{xy}} = 1$

$\Leftrightarrow x – \sqrt{xy} + y = \sqrt{xy}$

$\Leftrightarrow x- 2\sqrt{xy} + y = 0$

$\Leftrightarrow (\sqrt x – \sqrt y)^2 = 0$

$\Leftrightarrow \sqrt x – \sqrt y = 0$

$\Leftrightarrow \sqrt x = \sqrt y \Leftrightarrow x= y.$

Vậy $ \dfrac{x}{y} = 1. $

Bài 2.

a) Giải phương trình: $\dfrac{2x^2 (7-x)}{\sqrt{3-x}} = x(x-7)$

b) Giải hệ phương trình: $\left\{ \begin{array}{l} (x+3)(x-1)=(y-2)(x+3) \\ (x-1)\sqrt{y^2-5y+8}=(y-2)^2 \end{array} \right. $

Giải

a)  Điều kiện $ 3- x > 0 \Leftrightarrow x < 3. $

Ta có: $\dfrac{2x^2 (7-x)}{\sqrt{3-x}}= x(x-7)$

$\Leftrightarrow \dfrac{2x^2(7-x)}{\sqrt{3-x}} = \dfrac{x(x-7)\sqrt{3-x}}{\sqrt{3-x}}$

$\Rightarrow 2x^2(7-x) = x(x-7)\sqrt {3-x}$

$\Leftrightarrow x(x-7)(\sqrt{3-x} + 2x)= 0$

$\Leftrightarrow x = 0 \ (n) \text{ hoặc } x = 7 \ (l) \text{ hoặc } \sqrt{3-x} +2x = 0  \ (1)$

Giải $(1)$, ta được $ \sqrt{3-x} + 2x = 0 \Leftrightarrow \sqrt{3-x} = -2x $

$\Leftrightarrow \left\{ \begin{array}{l} x \le 0 \\ 3-x = 4x^2  \end{array} \right.$ $ \Leftrightarrow \left\{ \begin{array}{l}  x\le 0 \\  4x^2 + x – 3 = 0 \end{array}\right.$  $ \Leftrightarrow \left\{ \begin{array}{l} x\le 0 \\ x = \dfrac{3}{4} \text{ (l) } \text{ hoặc } x = -1 \text{ (n) } \end{array} \right. $

Vậy phương trình đã cho có nghiệm là $ S = \left\{ { – 1;0} \right\}$.

b) Giải hệ phương trình $\left\{ \begin{array}{l} (x+3)(x-1) = (y-2)(x+3) \ (1) \\ (x-1)\sqrt{y^2 – 5y + 8} =(y -2)^2 \ (2) \end{array} \right. $

Điều kiện: $y^2-5y+8 \ge 0 \Leftrightarrow y\in \mathbb{R}$

Giải $(1)$ Ta có $ (x+3)(x-1) = (y-2)(x+3) \Leftrightarrow (x+3)(x-1-y+2) = 0 $

$\Leftrightarrow (x+3)(x-y +1)= 0 \Leftrightarrow \left[ \begin{array}{l} x= -3\\ x = y -1 \end{array}\right.  $

Thay $ x = -3 $ vào $(2)$ ta được

$ (-3-1)\sqrt{y^2 – 5y + 8} =(y-2)^2 \Leftrightarrow -4\sqrt{y^2- 5y + 8} =(y -2)^2 \ \text{ (vô nghiệm)} . $

Vì $(y-2)^2 \ge 0 ;\ – 4\sqrt{y^2 – 5y + 8} < 0.$

Thay $ x = y -1 $ vào $(2)$ ta được

$ (y – 2)\sqrt{y^2 – 5y + 8} =(y-2)^2 \Leftrightarrow (y -2)(\sqrt{ y^2 – 5y + 8} – y + 2) = 0 $

$\Leftrightarrow \left[ \begin{array}{l}  y = 2\\  \sqrt{y^2 – 5y + 8} = y – 2 \ (3) \end{array} \right. $

Thay $ y = 2 \Rightarrow x = 1 $, ta có

$ (3) \Leftrightarrow \left\{ \begin{array}{l} y – 2\ge 0 \\  y^2 -5y + 8 =(y-2)^2 \end{array} \right. \Leftrightarrow \left\{ \begin{array}{l} y \ge 2 \\ y = 4 \ \text{(nhận)} \end{array} \right.  $

Với $ y =4 $ thì $ x = 3. $

Vậy hệ phương trình đã cho có nghiệm là: $ S = \left\{ {(1;2), \ (3;4)} \right\} $

Bài 3. Cho phương trình $x^2 -x +3m-11=0$ $(1)$

a) Với giá trị nào của $m$ thì phương trình $(1)$ có nghiệm kép? Tìm nghiệm đó.

b) Tìm $m$ để phương trình $(1)$ có hai nghiệm phân biệt $x_1$, $x_2$ sao cho

$$2017x_1 + 2018x_2 =2019.$$

Giải

a) Phương trình $(1)$ có nghiệm kép $\Leftrightarrow \left\{ \begin{array}{l} 1\ne 0 \text{ (hiển nhiên)} \\ \Delta = 0 \end{array} \right. $

$\Leftrightarrow 1-4(3m-11) =0 \Leftrightarrow 45-12m =0 \Leftrightarrow m=\dfrac{45}{12} = \dfrac{15}{4}$

Với $m=\dfrac{15}{4}$ thì phương trình $(1)$ trở thành:

$x^2-x+\dfrac{1}{4}=0 \Leftrightarrow x=\dfrac{1}{2}$

Vậy khi $m=\dfrac{15}{4}$ thì phương trình $(1)$ có nghiệm $x=\dfrac{1}{2}$.

b) Để phương trình $(1)$ có hai nghiệm phân biệt $x_1$, $x_2$ thì

$\Delta >0 \Leftrightarrow 45-12m >0 \Leftrightarrow m < \dfrac{15}{4}$

Theo định lý Viete, ta có: $\left\{ \begin{array}{l} S=x_1+x_2 = 1 \\ P=x_1x_2=3m-11 \end{array} \right. $

$2017x_1+2018x_2=2019 \Leftrightarrow 2017 \left( x_1 + x_2 \right) +x_2 =2019 $

$\Leftrightarrow 2017+x_2=2019 \Leftrightarrow x_2 = 2$

Mà $x_1+x_2 =1$ nên $x_1=-1$

Lại có $x_1x_2 = 3m-11 \Rightarrow 3m-11 = -2 \Rightarrow m=3$ (thỏa)

Vậy $m=3$ thì phương trình có hai nghiệm thỏa mãn đề bài.

Bài 4.

a) Đầu tháng $5$ năm $2018$, khi đang vào thu hoạch, giá dưa hấu bất ngờ giảm mạnh. Nông dân $A$ cho biết vì sợ dưa hỏng nên phải bán $30\%$ số dưa hấu thu hoạch được với giá $1500$ đồng mỗi kilôgam ($1500đ/kg$), sau đó nhờ phong trào “giải cứu dưa hấu” nên đã may mắn bán hết số dưa còn lại với giá $3500đ/kg$; nếu trừ tiền đầu tư thì lãi được $9$ triệu đồng (không kể công chăm sóc hơn hai tháng của cả nhà). Cũng theo ông $A$, mỗi sào đầu tư (hạt giống, phân bón,…) hết $4$ triệu đồng và thu hoạch được $2$ tấn dưa hấu. Hỏi ông $A$ đã trồng bao nhiêu sào dưa hấu?

b) Một khu đất hình chữ nhật $ABCD$ ($AB<AD$) có chu vi $240$ mét được chia thành hai phần gồm khu đất hình chữ nhật $ABNM$ làm chuồng trại và phần còn lại làm vườn thả để nuôi gà ($M$, $N$ lần lượt thuộc các cạnh $AD$, $BC$). Theo quy hoạch trang trại nuôi được $2400$ con gà, bình quân mỗi con gà cần một mét vuông của diện tích vườn thả và diện tích vườn thả gấp ba lần diện tích chuồng trại. Tính chu vi của khu đất làm vườn thả.

Giải

a) Giả sử ông $A$ đã trồng $x$ sào dưa hấu. ($x>0$)

Tổng số tiền ông $A$ thu được từ việc bán dưa hấu là:

$30\%x \cdot 1500 \cdot 2000 + 70\% x \cdot 3500 \cdot 2000 = 5800000x$ (đồng)

Tổng chi phí của ông $A$ là: $4000000x$ (đồng)

Ta có phương trình:

$5800000x-4000000x=9000000 \Leftrightarrow x=5$

Vậy ông $A$ đã trồng $5$ sào dưa hấu.

b) Gọi $x$, $y$ ($m$) lần lượt là chiều rộng và chiều dài của mảnh đất $ABCD$ ($x<y$).

Tổng diện tích của khu đất là: $2400+\dfrac{2400}{3} = 3200$

Ta có hệ phương trình:

$\left\{ \begin{array}{l} 2(x+y)=240 \\ xy=3200 \end{array} \right. \Leftrightarrow \left\{ \begin{array}{l} x+y=120 \\ xy=3200 \end{array}\right. $

Do đó $x$ và $y$ là hai nghiệm của phương trình:

$t^2 -120t+3200=0 \Rightarrow \left[ \begin{array}{l} t= 80 \\ t= 40 \end{array} \right. \Rightarrow \left\{ \begin{array}{l} x=40 \\ y=80 \end{array} \right. $

Suy ra $AB=40 m$, $AD=80m$, suy ra $NC=\dfrac{2400}{40}=60m$.

Vậy chu vi vườn thả là $2(40+60)=200m$.

Bài 5. Tứ giác $ABCD$ nội tiếp đường tròn $(T)$ tâm $O$, bán kính $R$; $\angle CAD = 45^\circ $, $AC$ vuông góc với $BD$ và cắt $BD$ tại $I$, $AD>BC$. Dựng $CK$ vuông góc với $AD$ ($K\in AD$), $CK$ cắt $BD$ tại $H$ và cắt $(T)$ tại $E$ ($E \ne C$)

a) Tính số đo góc $\angle COD$. Chứng minh các điểm $C$, $I$, $K$, $D$ cùng thuộc một đường tròn và $AC=BD$.

b) Chứng minh $A$ là tâm đường tròn ngoại tiếp tam giác $BHE$. Tính $IK$ theo $R$.

c) $IK$ cắt $AB$ tại $F$. Chứng minh $O$ là trực tâm tam giác $AIK$ và $CK \cdot CB = CF \cdot CD$.

Giải

a)

  • Ta có $\widehat{COD}=2\widehat{CAD}=2\times 45^\circ=90^\circ$ (góc ở tâm bằng hai lần góc nội tiếp cùng chắn cung $CD$).

$\widehat{CID}=\widehat{CKD}=90^\circ$

Suy ra $A$, $I$, $K$, $D$ cùng thuộc đường tròn đường kính $CD$.

  • $\triangle IAD$ có $\widehat{I}=90^\circ$, $\widehat{IAD}=45^\circ$ suy ra $\widehat{IDA}=45^\circ$, do đó $\triangle IAD$ vuông cân tại $I$.

Suy ra $IA=ID$ $(1)$.

  • $\widehat{CBI}=\widehat{IAD}=45^\circ$,

$\triangle ICB$ có $\widehat{CIB}=90^\circ$, $\widehat{CBI}=45^\circ$ suy ra $\widehat{ICB}=45^\circ$, do đó $\triangle ICB$ vuông cân tại $I$.

Từ đó suy ra $IC=IB$ $(2)$.

  • Từ $(1)$ và $(2)$ suy ra $IA+IC=IB+ID$, do đó $AC=BD$.

b)

  • Tứ giác $\triangle IHK$ có $\widehat{I}+\widehat{K}=90^\circ +90^\circ =180^\circ$.

Suy ra $AIHK$ nội tiếp.

Suy ra $\widehat{CHB}=\widehat{CAD}=45^\circ=\widehat{CBH}$.

Do đó $\triangle CBH$ vuông cân tại $C$ có $CI$ là đường cao nên cũng là đường trung trực đoạn thẳng $BH$.

Suy ra $AB=AH$. $(3)$

  • Ta có $\widehat{HAD}=\widehat{HCD}$ (cùng phụ $\widehat{ADC}$),

Mà $\widehat{HCD}=\widehat{DAE}$ nên $\widehat{HAD}=\widehat{DAE}$.

Suy ra $\triangle AKH = \triangle AKE$ (g.c.g).

Suy ra $AH=AE$ $(4)$

Từ $(3)$, $(4)$ ta được $AH=AE=AB$ nên $A$ là tâm đường tròn ngoại tiếp $\triangle BHE$.

  •  $\triangle AKC\backsim \triangle AID$ suy ra $AK.AD=AI.AC$

Do đó $\dfrac{AK}{AC}=\dfrac{AI}{AD}$.

Suy ra $\triangle AIK \backsim \triangle ACD$ suy ra $\dfrac{IK}{CD}=\dfrac{AK}{AC}=\cos \widehat{CAK}= \cos 45^\circ = \dfrac{1}{\sqrt{2}}$.

Mà $CD=\sqrt{CO^2 + OD^2}=R\sqrt{2}$.

Do đó $IK=R$.

c)

  • Ta có $\left\{ \begin{array}{l} IA=ID\\ OA=OD \end{array} \right. $ suy ra $IO$ là trung trực $AD$, do đó $IO\perp AD$. $(5)$

$\left\{ \begin{array}{l} KC=KA \\ OC=OA \end{array} \right. $ suy ra $KO$ là trung trực $AC$, do đó $KO\perp AC$.  $(6)$

Từ $(5)$, $(6)$ suy ra $O$ là trực tâm $\triangle AIK$.

  • Ta có $\widehat{CAF}=\widehat{CDB}$ (cùng bằng nửa số đo cung $CB$).

$\widehat{CDB}=\widehat{CKF}$ (Tứ giác $CIKD$ nội tiếp).

Suy ra $\widehat{CAF}=\widehat{CKF}$, do đó tứ giác $CKAF$ nội tiếp.

Từ đó suy ra $\widehat{CFA}=180^\circ – \widehat{CKA}=90^\circ$.

  • Xét tam giác $\triangle CBF$ và $\triangle CKD$,

$\begin{array}{l} \widehat{CFB}=\widehat{CKD}=90^\circ\\ \widehat{CBF}=\widehat{CDK}\text{ (tứ giác $ABCD$ nội tiếp)} \end{array}$

Suy ra $\triangle CBF\backsim \triangle CDK$.

Do đó $\dfrac{CB}{CD}=\dfrac{CF}{CK}$

Suy ra $CB.CK=CD.CF$.

Đáp án đề thi học kì 1 môn toán 10 năm học 2018 trường PTNK – Cơ sở 2

Bài 1. Giải các phương trình sau:
a)$\sqrt{7x+2} = 1 + \sqrt{4x+1}$
b) $\left| x^2-x-1 \right|+3= 2x$
Bài 2. 

a) Tìm $a$, $b$, $c$ biết parabol $(P):y= ax^2 + bx +c$ đi qua điểm $A(1;-1)$ và có đỉnh $I(-1; -5)$.
b) Tìm $m$ để phương trình $(x-1)\left( \sqrt{x+m}-1 \right) =0$ có hai nghiệm là độ dài các cạnh góc vuông của một tam giác vuông với cạnh huyền có độ dài bằng 3.
Bài 3. Cho hệ phương trình
$$\left\{ \begin{array}{l}
(1-2m)x +4y = 4m^2 + 4m +3 \\
mx + 2(m-1)y=-m-2
\end{array} \right. \quad (I) $$
Chứng minh khi $m$ nhận giá trị bất kì trên $\mathbb{R}$, hệ $(I)$ luôn có nghiệm duy nhất. Giả sử $(x_0, 1)$ là nghiệm của hệ $(I)$. Tìm $x_0$.
Bài 4. Cho góc $\alpha$ thỏa $\tan \left( \alpha + \dfrac{\pi}{3} \right) = -\dfrac{ 3\sqrt{3}}{5}$. Tính giá trị của biểu thức:
$$ P=\dfrac{\cos ^3 \alpha + 2\sin \alpha \cdot \cos ^2 \alpha}{\sin ^2 \alpha \cdot \cos \alpha + \sqrt{3} \sin ^3 \alpha }$$
Bài 5. Cho tam giác $ABC$ có $AB=3a$, $AC=6a$, $BC=7a$.
a) Tính $\overrightarrow{AB} \cdot \overrightarrow{AC}$ và $\cos A$.
b) Gọi $M$, $N$ là hai điểm được xác định bởi $\overrightarrow{AM} = \dfrac{2}{3} \overrightarrow{AB}$, $\overrightarrow{AN} = -\dfrac{3}{2} \overrightarrow{AC}$, tính $MN$ theo $a$.

Bài 6. Trong mặt phẳng $Oxy$, cho tam giác $ABC$ có $A(1;3)$, $B(6;-2)$.

a) Tìm tọa độ điểm $C$ sao cho $G(1;1)$ là trọng tâm của tam giác $ABC$.
b) Tìm tọa độ chân đường vuông góc kẻ từ $M(4;3)$ đến đường thẳng $AB$.

Hết

Lời giải

 

Bài 1. 

a) Nghiệm của phương trình: $x=2$.
b) $\left| x^2-x-1 \right| +3= 2x$
$\Leftrightarrow \left| x^2-x-1 \right| = 2x-3 \quad \left( x \ge \dfrac{3}{2} \right) $
$\Leftrightarrow \left[ \begin{array}{l}
x^2-x-1 = 2x-3 \\\\
x^2-x-1=3-2x
\end{array} \right. $
Từ đó suy ra nghiệm của phương trình: $x=2$ hoặc $x=\dfrac{\sqrt{17}-1}{2}$

Bài 2.

a) $P$ qua điểm $A(1;-1)$ nên $-1=a+b+c$.

$(P)$ có đỉnh $I(-1;-5)$ nên $-5=a-b+c$ và $-\dfrac{b}{2a}= -1$.

Từ đó suy ra $P: y= x^2 + 2x-4$.
b) Điều kiện: $x\ge -m$

Từ phương trình suy ra: $\left[ \begin{array}{l}
x= 1 \\\\
x= 1-m
\end{array} \right. $

Để hai nghiệm là độ dài các cạnh góc vuông của tam giác vuông có cạnh huyền bằng 3 thì: $1^2 + (1-m)^2 =3^2$ $\Rightarrow \left[ \begin{array}{l}
m= 1+2\sqrt{2} \\\\
m= 1-2\sqrt{2} \quad \text{(loại vì } x\ge -m)
\end{array} \right. $

Vậy $m=1+2\sqrt{2}$.

Bài 3. 

$D= \left| \begin{array}{*{20}{c}}
{1-2m}&{4}\\\\
{m}&{2(m-1)}
\end{array} \right| = (1-2m)(2m-2)-4m = -4m^2+2m-2 $

$D_x = \left| \begin{array}{*{20}{c}}
{4m^2+4m+3}&{4}\\\\
{-m-2}&{2(m-1)}
\end{array} \right| = 8m^3+2m+2 $

$D_y= \left| \begin{array}{*{20}{c}}
{1-2m}&{4m^2+4m+3}\\\\
{m}&{-m-2}
\end{array} \right| = -4m^3-2m^2-2 $

$D=0 \Leftrightarrow -4m^2+2m-2 =0$ (vô nghiệm).

Suy ra $D \ne 0$ với mọi $m \in \mathbb{R}$

Vậy với mọi $m \in \mathbb{R}$ thì hệ $(I)$ luôn có nghiệm duy nhất.

Khi $(x_0;1)$ là nghiệm của hệ $(I)$ thì $y=\dfrac{D_y}{D}=\dfrac{-4m^3-2m^2-2}{-4m^2+2m-2}=1 \Leftrightarrow m=0$

Khi đó $x=\dfrac{D_x}{D}= \dfrac{2}{-2}=-1$

Bài 4.

$\tan\left( \alpha + \dfrac{\pi}{3} \right) =\dfrac{\tan \alpha + \tan \dfrac{\pi}{3}}{1-\tan \alpha \cdot \tan \dfrac{\pi}{3}} = -\dfrac{3\sqrt{3}}{5} \Leftrightarrow \tan \alpha = 2\sqrt{3}$

$P=\dfrac{\cos ^3 \alpha + 2\sin \alpha \cdot \cos ^2 \alpha}{\sin ^2 \alpha \cdot \cos \alpha + \sqrt{3} \sin ^3 \alpha } = \dfrac{1+2\tan \alpha}{\tan ^2 \alpha + \sqrt{3} \tan ^3 \alpha} = \dfrac{1+4\sqrt{3}}{84}$

Bài 5. 

a) $\overrightarrow{AB} \cdot \overrightarrow{AC} = -2a^2$; $\cos A = \dfrac{-2a^2}{3a\cdot 6a}= -\dfrac{1}{9}$
b) $\overrightarrow{MN} = \overrightarrow{AN} – \overrightarrow{AM} = -\dfrac{3}{2} \overrightarrow{AC} – \dfrac{2}{3} \overrightarrow{AB} \Rightarrow MN^2=\overrightarrow{MN}^2 = \ldots = 81a^2 \Rightarrow MN = 9a$

Bài 6. 

a) $C(-4;2)$
b) Gọi chân đường vuông góc hạ từ $M$ đến $AB$ là $H(x,y)$

Ta có: $\left\\{ \begin{array}{l}
\overrightarrow{MH} \bot \overrightarrow{AB} \\\\
\overrightarrow{AH} \parallel \overrightarrow{AB}
\end{array} \right. $

Từ đó suy ra: $H\left( \dfrac{5}{2}; \dfrac{3}{2} \right) $

Đáp án đề thi HK1 lớp 11 trường PTNK năm học 2017 – 2018

Bài 1. Giải các phương trình sau:
a)  $ 2\cos ^2 \dfrac{x}{2}+\sqrt{3}\sin x=1+2\sin 3x $
b) $ 3 \tan^2 x+4\tan x+4\cot x+3\cot^2 x+2=0 $

Bài 2. Gọi S là tập tất cả các số tự nhiên gồm 4 chữ số phân biệt được chọn từ các số 1;2;3;4;5;6;7. Lấy ngẫu nhiên một số từ S. Tính xác suất để lấy được số có mặt chữ số 6.

Bài 3. Trong khai triển của $ \left(2x^3-\dfrac{3}{x^2}\right)^n $ với $ n $ là số nguyên dương thỏa $ 2C_{n+6}^{5}=7A_{n+4}^3, $ tìm số hạng không chứa $ x? $

Bài 4. Tìm số hạng đầu và công sai của cấp số cộng $ (u_{n}) $ biết rằng công sai của $ (u_{n}) $ là số nguyên dương và
$u_{1}+u_{3}+u_{5}=15, \dfrac{1}{u_{1}}+\dfrac{1}{u_{3}}+\dfrac{1}{u_{5}}=\dfrac{59}{45} $.

Bài 5. Trong mặt phẳng tọa độ Oxy, cho điểm $ I(2;-5) $ và đường thẳng $ d:3x-2y+3=0. $ Viết phương trình đường thẳng $ d’ $ là ảnh của $ d $ qua phép đối xứng tâm $ I. $

Bài 6. Cho hình chóp $ S.ABCD $ có đáy $ ABCD $ là hình thang có $ AD $ là đáy lớn, $ AD=2BC. $ Gọi $ O $ là giao điểm của $ AC $ và $ BD. $ Gọi $ G_{1},G_{2} $ lần lượt là trọng tâm $ \Delta SCD, \Delta SAB, \ E $ là trung điểm $ SD. $
a)  Mặt phẳng $ (BCE) $ cắt $ SA $ tại $ F. $ Chứng minh: $ F $ là trung điểm $ SA. $
b) Chứng minh $ G_{1}G_{2} \parallel (SAD) $
c) Chứng minh $ (OG_{1}G_{2}) \parallel (SBC) $
d) Gọi $ M $ là điểm trên cạnh $ AB $ sao cho $ AB=4AM. $ Mặt phẳng $ (P) $ qua $ M $ và song song với $ BC, SD. $ Xác định thiết diện của hình chóp với mặt phẳng $ (P). $ Thiết diện là hình gì?

Hết

Đáp án

[userview]

Bài 1.

a) Phương trình tương đương với
$$
\begin{aligned}
& \cos x+\sqrt{3} \sin x=2 \sin 3 x \\
\Leftrightarrow & \frac{1}{2} \cos x+\frac{\sqrt{3}}{2} \sin x=\sin 3 x \\
\Leftrightarrow & \sin \left(x+\frac{\pi}{6}\right)=\sin 3 x \\
\Leftrightarrow x+\frac{\pi}{6}=3 x+k 2 \pi \text { hoặc } x+\frac{\pi}{6}=\pi-3 x+k 2 \pi \\
\Leftrightarrow x=\frac{\pi}{12}+k \pi \text { hoặc } x=\frac{5 \pi}{24}+\frac{k \pi}{2}, k \in \mathbb{Z}
\end{aligned}
$$

Bài 2. Gọi $\overline{a b c d}(a \neq 0)$ là số tự nhiên có 4 chữ số phân biệt được chọn từ các chữ số 1,2,3,4,5,6,7 .
$\overline{a b c d}:$ Có $A_{7}^{4}=840$ số tự nhiên có 4 chữ số phân biệt được chọn từ các chữ số 1,2,3,4,5,6,7
$\Longrightarrow|\Omega|=840$Gọi A là biên có sao cho số dược lậy là một số có mặt chữ số $6 .$
$$
|A|=4 . A_{6}^{3}=480 \Longrightarrow P(A)=\frac{|A|}{|\Omega|}=\frac{4}{7}
$$

Bài 3. 

\begin{aligned}
&2 C_{n+6}^{5}=7 A_{n+4}^{3} \Longleftrightarrow 2 \cdot \frac{(n+6) !}{5 !(n+1) !}=7 \cdot \frac{(n+4) !}{(n+1) !} \Longleftrightarrow \frac{(n+6) !}{(n+4) !}=420 \Longleftrightarrow(n+6)(n+5)=\\
&420 \Longleftrightarrow n^{2}+11 n-390=0 \Longleftrightarrow\left[\begin{array}{l}
n=15 \\
n=-26
\end{array} \Longleftrightarrow n=15(\text { vì n là số tự nhiên })\right.\\
&\text { Công thức } \mathrm{SHTQ}: T_{k+1}=C_{15}^{k} \cdot\left(2 x^{3}\right)^{15-k} \cdot\left(-\frac{3}{x^{2}}\right)^{k}=C_{15}^{k} \cdot 2^{15-k} \cdot(-3)^{k} \cdot x^{45-5 k}\\
&\text { Để số hạng không chứa } x \Longleftrightarrow 45-5 k=0 \Longleftrightarrow k=9 \text { . }\\
&\text { Vậy số hạng không chứa } \mathrm{x}: T_{10}=C_{15}^{9} .2^{6} \cdot(-3)^{9}=-6304858560 \text { . }
\end{aligned}

Bài 4. $\left\{\begin{array}{l}
u_{1}+u_{3}+u_{5}=15(1) \\
\frac{1}{u_{1}}+\frac{1}{u_{3}}+\frac{1}{u_{5}}=\frac{59}{45}(2) \end{array} \right.$
$(1) \Longleftrightarrow 3 u_{3}=15 \Longleftrightarrow u_{3}=5 $
$(2) \Longleftrightarrow \frac{1}{u_{1}}+\frac{1}{5}+\frac{1}{u_{5}}=\frac{59}{45} \Longleftrightarrow \frac{1}{u_{1}}+\frac{1}{u_{5}}=\frac{10}{9} $

$\Longleftrightarrow 9\left(u_{1}+u_{5}\right)=10 u_{1} u_{5} $

$\Longleftrightarrow 9.2 u_{3}= 10\left(u_{3}-2 d\right)\left(u_{3}+2 d\right)$

$\Longleftrightarrow 90=10\left(u_{3}^{2}-4 d^{2}\right)=25-4 d^{2}=9 $

$\Longleftrightarrow d^{2}=4$

$\Longleftrightarrow d=2(\text{vì} d>0) $
$u_{3}=5 \Longleftrightarrow u_{1}+2 d=5 \Longleftrightarrow u_{1}=5-2 d=1$.
và $u_{1}=1,d=2$

Bài 5. 

Gọi $M^{\prime}\left(x^{\prime} ; y^{\prime}\right)$ là ảnh của $\mathrm{M}$ qua phép đối xứng tâm $\mathrm{I} \Longleftrightarrow \mathrm{I}$ là trung điểm của $\mathrm{MM}^{\prime} \Longleftrightarrow$
$$
\left\{\begin{array}{l}
x_{I}=\frac{x_{M}+x_{M^{\prime}}}{2} \\
y_{I}=\frac{y_{M}+y_{M^{\prime}}}{2}
\end{array} \Longleftrightarrow\left\{\begin{array}{l}
4=x+x^{\prime} \\
-10=y+y^{\prime}
\end{array} \Longleftrightarrow\left\{\begin{array}{l}
x=4-x^{\prime} \\
y=-10-y^{\prime}
\end{array}\right.\right.\right.
$$
Ta có: $3 x-2 y+3=0 \Longleftrightarrow 3\left(4-x^{\prime}\right)-2\left(-10-y^{\prime}\right)+3=0 \Longleftrightarrow 12-3 x^{\prime}+20+2 y^{\prime}+3=0 \Longleftrightarrow$
$3 x^{\prime}-2 y^{\prime}-35=0$
Vậy M’ thuộc dường thẳng d’:3x-2y-35=0.
Vậy ảnh của đường thẳng d qua phép đối xứng tâm I là đường thẳng $\mathrm{d}^{\prime}: 3 \mathrm{x}-2 \mathrm{y}-35=0 .$

Bài 6. 

a) Ta có: $C \in(S A C) \cap(B C E)(1)$.
Trong $(S B D)$ gọi $\mathrm{K}$ là giao diểm của $\mathrm{SO}$ và $\mathrm{BE}$ mà $S O \subset(S A C), B E \subset(B C E)=K \in$
$(S A C) \cap(B C E)(2)$
$(1)(2) \Longrightarrow C K=(S A C) \cap(B C E)$
Trong $(S A C)$ gọi $\mathrm{F}$ là giao điểm của $\mathrm{SA}$ và $\mathrm{CK}$ mà $\mathrm{CK} \subset(B C E)=F=\operatorname{SAn}(B C E) .$ $\mathrm{Vi} A D \| B C=\frac{O C}{O A}=\frac{O B}{O D}=\frac{B C}{A D}=\frac{1}{2} \Longleftrightarrow \frac{C O}{C A}=\frac{B O}{B D}=\frac{1}{3}$
Xét $\triangle S O D$ : Áp dụng định lý Menelaus với 3 điểm $\mathrm{B}, \mathrm{K}, \mathrm{E}$ thẩng hàng ta có:
$\frac{C O}{C A} \cdot \frac{K S}{K O} \cdot \frac{F A}{F S}=1 \Longleftrightarrow \frac{F A}{F S}=1 \Longleftrightarrow \mathrm{F}$ là trung điẻm $\mathrm{SA}$
b) Trong (SAB), goi P là giao điểm của $S G_{1}$ và AB. Vì $G_{1}$ là trọng tâm của $\triangle S A B=P$
là trung điểm của AB.

Trong (SCD), gọi P là giao điểm của $S G_{2}$ và CD. Vì $G_{2}$ là trọng tàm của $\triangle S C D=\mathrm{Q}$
là trung điểm của CD. Xét $\triangle S P Q$ ta có: $\frac{S G_{1}}{S P}=\frac{2}{3}=\frac{S G_{2}}{S Q}=G_{1} G_{2} \| P Q(3)$

Xét hình thang ABCD ta có: PQ là đường trung bình của hình thang ABCD (do P,Q làn
lượt là trung điểm của $\mathrm{AB}, \mathrm{CD} \Longrightarrow P Q \| A D(4)$
$$
\text { Tì }(3)(4)=G_{1} G_{2}\left\|A D, \operatorname{mà} \mathrm{AD} \subset(\mathrm{SAD})=G_{1} G_{2}\right\|(S A D)
$$
c) Ta có: $G_{1} G_{2} \| A D$ mà $A D\left\|B C=G_{1} G_{2}\right\| B C=G_{1} G_{2} \|(S B C)(5)$
Trong (SAB), gọi H là giao điểm của $A G_{1}$ và $\mathrm{SB}$. Vì $G_{1}$ là trọng tần của $\triangle S A B=\mathrm{H}$
là trung điểm của $\mathrm{SB}$. Xét $\triangle H A C$ ta có: $\frac{A O}{A C}=\frac{2}{3}=\frac{A G_{1}}{A H}=O G_{1}\left\|C H \operatorname{mà} C H \subset(S B C)=O G_{1}\right\|(S B C)(6)$
Tì $(5)(6)=\left(O G_{1} G_{2}\right) \|(S B C)$
d) Ta có: $M \in(P) \cap(A B C D) \operatorname{mà}(P)\left\|B C=(P) \cap(A B C D)=x M x^{\prime}\right\| B C$.
Trong (ABCD), gọi N là giao diểm của xMx’ và CD.
Ta có: $N \in(P) \cap(S C D) \operatorname{mà}(P)\left\|S D=(P) \cap(S C D)=y N y^{\prime}\right\| S D$
Trong (SCD) gọi I là giao diểm của yNy’ và SC.
Ta có: $I \in(P) \cap(S B C) \operatorname{mà}(P)\left\|B C \Longrightarrow(P) \cap(S B C)=t I t^{\prime}\right\| B C .$
Trong (SBC), gọi J là giao điểm của tIt’ và SB. $((P) \cap(A B C D)=M N$
$\Longrightarrow$ thiệt diê
Ta có: $M N\|I J\| A D=M N I J$ là hình thang.

[/userview]